Vous êtes sur la page 1sur 44

Medsurg 2 06S6691R69 1 A nurse assessing a client diagnosed with bronchitis is likely to find: Incorrect: This breath sound pattern

is characteristic of pneumothorax.Incorrect: This assessment finding is associated with pleural effusion. Correct: With bronchitis, breath sounds are in the normal-to-decreased range, and wheezing is common due to narrowing of the bronchial lumen.Incorrect: This is common in clients who have emphysema and is usually heard with prolonged expiration. totally absent breath sounds. decreased to absent breath sounds. normal to decreased breath sounds. a decreased intensity of breath sounds. 2 A client's serum sodium level is 130 mEq/L; he is also mildly dehydrated. The nurse checks the provider's orders and then prepares to give this client: Incorrect: The client's fluid volume is already low, and diuretics will further decrease it. Thiazide diuretics also remove sodium from the body, and this client's sodium level is already too low.Incorrect: The purpose of polystyrene sulfonate (Kayexalate) administration is to remove excess potassium from the gastrointestinal tract. It often causes diarrhea that consequently promotes additional loss of fluid volume.Tap water enemas promote the loss of sodium from the gastrointestinal tract.Correct: The client's sodium level is low. Beef broth is high in sodium content and might help correct the hyponatremia. The additional fluid intake might also help correct the client's dehydration. thiazide diuretics. sodium polystyrene sulfonate (Kayexalate). a tap water enema. beef broth. 3 In designing a presentation about diabetes management for a group of clients, a nurse plans to emphasize factors that impact blood glucose control. One of these factors is that: Correct: To reach and maintain target glucose levels, it is important to exercise daily at moderate and consistent levels. Regular exercise helps reduce the need for medication, lower triglyceride and low-density lipoprotein levels, reduce blood pressure, and promote circulation.Incorrect: Diabetic management is susceptible to many variables, therefore, clients requiring insulin must check their blood glucose levels daily as prescribed by the provider, typically 4 times a day, before each meal and at bedtime.Incorrect: The incidence of diabetic retinopathy is high and is already underway and treatable before clients experience vision problems. Therefore, people who have diabetes are encouraged to get regularly scheduled eye exams for early detection of problems.Incorrect: UTIs are common in clients who have diabetes and must be treated quickly and aggressively, but not

prophylactically. regular, consistent, and moderate exercise on a daily basis is absolutely essential. checking blood glucose before meals is optional once an insulin regimen is established. regular eye exams are necessary as soon as clients start experiencing vision difficulties. antibiotics are prescribed prophylactically to prevent urinary tract infections (UTIs). 4 A nurse is teaching family caregivers how to safeguard an 80-year-old client with Alzheimer's disease who is living with them in their home. To promote client safety, the nurse emphasizes that the client should always wear: Incorrect: Caregivers should avoid using physical restraints such as protective vests on wandering clients because this can make them feel even more restless and agitated.Incorrect: GPS devices are now becoming available to track clients who have Alzheimer's disease. However GPS devices and accompanying support services are costly, making this an inappropriate choice for many family caregivers.Incorrect: Caregivers should avoid using physical restraints such as leather restraining belts on wandering clients because they can make them feel even more restless and agitated.Correct: Clients who have Alzheimer's disease are prone to restlessness and wandering. If clients wander from home, they can be identified readily if they are wearing identification badges or bracelets and returned home safely. a vest restraint. a global positioning system (GPS) device. a leather restraining belt. an identification badge or bracelet. 5 A nurse recognizes and notifies the provider of a hemodynamic change in a client's status when assessment data reveal a(n): Incorrect: Although this could be an early sign of infection, this body temperature change is not significant enough at this point to affect the client's hemodynamic status.Incorrect: A dropping oxygen saturation is a reflection of impaired gas exchange, however this change still has the client within the normal range. Frequent reassessment is indicated.Incorrect: An increase in heart rate could reflect hemodynamic change, but this increase is not significant enough at this point. The nurse must evaluate the client for any possible causes for the rate increase, such as increased activity, anxiety, or medication adverse effects before concluding, based on this one parameter, that the client's hemodynamic status is altered.Correct: Depending on the client's baseline blood pressure and the time frame in which this decrease occurred and various other factors, a falling blood pressure, especially when the pulse rate is increasing, reflects a reduction in cardiac output. This is a hemodynamic change that must be reported immediately. temperature change from 99.2 to 99.8 F (37.3 to 37.7 C). O2 saturation change from 99% to 96%.

heart rate change from 78 to 86 beats per minute. blood pressure change from 116/58 to 84/46. 6 A client admitted with chronic renal failure has a serum creatinine of 4.6 mg/dL, a serum potassium of 6.5 mEq/dL, and a serum sodium of 132 mEq/L. In response to these laboratory results, which is the nurse's highest-priority intervention? Incorrect: Weighing a client with renal failure is important, but it is not the highest priority based on these lab values.Incorrect: Clients who have end-stage renal disease, the stage in which electrolyte imbalances are commonly seen, is probably already receiving hemodialysis. Correct: Hyperkalemia is a potentially life-threatening emergency, as it can trigger ventricular dysrhythmias and cardiac arrest. The nurse's first priority is to assess the client's apical pulse for rate and regularity or irregularity before taking any other action.Incorrect: The client's sodium level is within normal limits. Seizure precautions would be a priority with an excessively low serum sodium level. Weighing the client immediately Arranging for immediate hemodialysis Auscultating the client's apical pulse Implementing seizure precautions 7 A nurse is setting up a sterile field in preparation for a bedside surgical procedure. Before beginning, she performs hand hygiene thoroughly according to institutional policy. Prioritize the order in which the nurse then performs the following steps, indicating in the correct order the 4 numbers corresponding to the steps, with no spaces or punctuation between them. 1. Pouring a sterile antiseptic solution into a bowl on the sterile field 2. Putting on sterile gloves using the appropriate gloving technique 3. Performing hand hygiene again using the method the facility specifies 4. Opening the package to drape the sterile field, letting the edges of the wrapping drop down Correct: When setting up a sterile field, the nurse first drapes the sterile field by opening the package and letting the edges of the wrapping drop down and away. She is careful to touch only the outside surface of the drape. Next, the nurse prepares a sterile antiseptic solution. She removes the lid or cap and places it sterile side up on a clean surface. With the bottle in the palm of her hand and 4 to 6 inches above the bowl or other container in the sterile field, she carefully pours the liquid, making every effort to avoid splashing it. Next, she washes her hands, then opens and removes the outer packaging of

the gloves. She places the inner packaging on a clean, dry surface and uses either the openor closed-glove technique for applying the sterile gloves.Incorrect: When setting up a sterile field, the nurse first drapes the sterile field by opening the package and letting the edges of the wrapping drop down and away. She is careful to touch only the outside surface of the drape. Next, the nurse prepares a sterile antiseptic solution. She removes the lid or cap and places it sterile side up on a clean surface. With the bottle in the palm of her hand and 4 to 6 inches above the bowl or other container in the sterile field, she carefully pours the liquid, making every effort to avoid splashing it. Next, she washes her hands, then opens and removes the outer packaging of the gloves. She places the inner packaging on a clean, dry surface and uses either the open- or closed-glove technique for applying the sterile gloves. 8 A 62-year-old client who has chronic kidney disease has just been admitted. When planning care for this client, the nurse will make sure this client's meals include dietary restriction of: Incorrect: Apples contain fiber and carbohydrates, and are lower in potassium than some other fruits that might be restricted when potassium intake must be kept low.Incorrect: Rice contains carbohydrates, some fiber, and is relatively low in protein. It could safely be included in the diet of a client who has chronic kidney disease.Correct: For clients who have chronic renal failure, phosphate intake should be limited to 1,000 mg per day. Dairy products such as milk, cheese, and yogurt are especially high in phosphate content. Some cheeses are also high in sodium, also restricted for some clients which chronic renal disease.Incorrect: Cucumbers are low in all of the nutrients that might be restricted in a diet for clients who have chronic renal disease. Restricted vegetables include broccoli, carrots, squash, potatoes, and pinto beans. apples. rice. cheese. cucumbers. 9 A nurse is providing discharge instructions for a client previously diagnosed with hypertension who has been poorly controlled prior to admission. The client tells the nurse that he has been unsuccessful in complying with his provider's recommended lifestyle changes, especially diet. When instructing the client about sodium restriction, the nurse includes which information likely to be especially helpful for this client? Incorrect: The National Institutes of Health in the United States and the American Heart Association recommend that clients follow the DASH diet (Dietary Approaches to Stop Hypertension). Following this plan helps clients reduce blood pressure by eating foods that are low in saturated fat, total fat, and cholesterol, and are high in fiber. Correct: A major factor in complying with a reduced sodium diet is adjusting to the change in taste when foods are prepared without salt. This client might be encouraged if he realizes that, given time, he has a good chance of adjusting to and actually enjoying low-sodium food choices.Incorrect: For some clients, the provider will prescribe lifestyles changes to see if they will be sufficient for controlling hypertension. If not, antihypertensive

medication is prescribed. However, clients taking blood pressure medications must still make lifestyle changes, including sodium restriction, to improve control and enhance the effect of the medication. In fact, with lifestyle changes, the dosage of the medication might eventually be able to be reduced.Incorrect: This might be true for some clients, however, people with hypertension respond differently to sodium restriction. Some are salt-sensitive and some are salt-resistant. Certainly with good dietary compliance, stepping down to a lower dosage might be trialed, but some clients, despite sodium restriction, will require antihypertensive medication on a lifelong basis. He should focus his diet primarily on sodium restriction with less emphasis on fiber and fat. It will probably take 2 to 3 months for his sense of taste to adjust to changes in salt intake. If he takes his newly prescribed medication, he can maintain his usual sodium intake. As long as he reduces his sodium intake as prescribed, he will no longer need his medication. 10 A 60-year-old client has developed fluid volume excess as a result of hypotonic overhydration. The nurse caring for this client promotes the elimination of edema-related fluid by: Incorrect: This is an important nursing intervention for clients who have edema, because edematous tissue is more susceptible to skin breakdown than normal tissue is. However, turning the client will not necessarily promote diuresis.Incorrect: Although drinking water helps to eliminate waste products from the body, this client's imbalance cannot be corrected, and might be worsened, by giving the client more fluid to drink. Overhydration is treated with fluid restriction and whatever medical or pharmacologic treatment is prescribed to help the client release excess fluid, such as diuretic therapy. The goal is to treat the cause of the client's overhydration and fluid restriction will be prescribed (or not) accordingly.Correct: Bed rest seems to promote diuresis, probably because it allows less venous pooling. With less pooled blood, circulating blood volume increases, thus improving renal perfusion.Incorrect: This is an essential intervention for this client, but it will not affect the actual mobilization of edematous fluid. It will help the nurse track the therapeutic benefits of the prescribed interventions as the client's outcome significantly exceeds intake. turning and repositioning the client frequently. increasing oral fluid intake to mobilize edema fluid. facilitating regular periods of bed rest for the client. monitoring the client's fluid intake and output closely. 11 A nurse is planning care for an 88-year-old client identified via Braden scale as at high risk for pressure ulcer development. A key nursing action the nurse will take for this client is: Correct: Correctly positioning pillows to relieve pressure on bony prominences is

called the bridging technique. An example is placing pillows superior and inferior to the sacrum to relieve sacral pressure.Incorrect: Semi-Fowler's is a poor choice for this client, because of the pressure the position exerts on the sacral area and the buttocks. Additionally, the client in semi-Fowler's is likely to slide down in bed, subjecting her skin to shearing forces (one layer of tissue sliding over another). Shearing forces also result when a client is pulled up improperly in bed. Recumbent positioning is preferable to semi-Fowler's for a client at risk for pressure ulcers.Incorrect: The nurse must avoid massaging any skin areas reddened as a result of pressure, because this action might further damage any traumatized skin, capillaries, and deep tissues.Incorrect: Digging the heels and ankles into the mattress puts the client at risk for pressure ulcers of the skin over the heels and the ankles. Clients at risk for skin breakdown can be moved in bed in any number of safer ways, such as using a draw sheet lift, having the client use her forearms and not her elbows to assist with moving up, or having the client use an overhead trapeze to assist with movement. using the bridging technique when positioning the client in bed. positioning her primarily in semi-Fowler's, alternating with side-lying. frequently massaging any erythematous areas detected on skin assessments. teaching the client to help move up in bed by using her heels and elbows to alleviate sacral pressure. 12 A nurse is caring for a client who is 2 days postoperative with a newly created tracheostomy. After assessing the client's vital signs, the nurse's priority intervention in caring for this client is: Incorrect: This is information the client and his family will need to learn before discharge to home. The sooner the nurse begins to teach them, the more time there will be to ensure proper learning and demonstrations. However, just after the nurse's assessment of the client's vital signs, teaching would not take highest priority.Incorrect: The nurse should encourage the client to cough to expectorate secretions. If the client's tracheostomy has an inner cannula, it will have a locking device to hold it in place. In most cases, coughing will not dislodge it.Incorrect: This is not a priority in the immediate postoperative period. The nurse should offer the client going home with a tracheostomy information about appropriate community resources as part of discharge planning and teaching, but after first discussing his needs. That way, the appropriate resources can be selected.Correct: Clients who have a tracheostomy, especially newer postoperative clients, often have difficulty expectorating sputum and need frequent suctioning. The tracheostomy is new to them and they are not able to communicate in the manner they were used to. Even if they are experiencing some symptoms, such as mild dyspnea or restlessness, they might be unable to communicate it to the nurse. Thus frequent assessments of respiratory status, including checking for an SpO2 below 90%, are essential. starting to teach the client and his family how to perform tracheostomy care. discouraging the client from coughing to avoid dislodging the inner cannula. familiarizing the client and his family with available community resources. assessing respiratory status, such as by checking peripheral oxygenation. 13

An 82-year-old client is diagnosed with extensive herpes zoster lesions appearing the previous day and covering the left side of his thoracic region. The nursing diagnosis of highest priority for this client at this time is: Incorrect: Without proper attention to preventing the transmission of bacteria to open lesions and the dissemination of virus-containing vesicle fluid, secondary bacterial infection and spreading of the viral infection are certainly risks. However, this is not the highest priority nursing diagnosis for this client at this time.Correct: The herpes zoster virus causes an acute, usually unilateral inflammation of a particular dorsal root ganglion in an area called a dermatome. The eruptions most often appear in the thoracic region and are preceded and accompanied by sharp, burning, stabbing pain that is severe in some clients. Relieving the client's pain and discomfort is the highest nursing priority at this time, and adequate pain control also helps prevent persistent pain patterns.Incorrect: Pruritus from open herpes zoster vesicles is common, and scratching the lesions further exposes injured tissue to infection. This is an important nursing diagnosis for some clients who have herpes zoster, and requires measures to soothe itchy skin, such as wet dressings, but this is not the highest priority nursing diagnosis for this client at this time.Incorrect: Herpes zoster commonly affects immunocompromised clients, such as those of advanced age or those who have an immune disorder as a result of a virus such as HIV or a treatment such as chemotherapy. Medical management of this client might include a determination of any underlying causes of impaired immunity and appropriate treatment to attempt to prevent future opportunistic infection, but this is not the nurse's highest care priority for this client at this time. risk for secondary infection due to tissue destruction. acute pain related to vesicular eruption along sensory nerves. impaired skin integrity related to pruritus from herpes zoster lesions. altered protection due to immunosuppression evidenced by extensive viral skin lesions. 14 A client reports that his colostomy pouch is always "inflated like a balloon." Which action should the nurse take? Correct: This is the first step in identifying a problem that would have a relatively simple solution. If the client's diet contains foods and beverages that are gas-producing, and he either consumes several of these items or any of them in plentiful amounts, cutting back on those items might solve the problem of excess gas being expelled via the stoma.Incorrect: Though this is a correct action, the nurse failed to assess the reason for the increased gas production by the client.Incorrect: This is not a correct intervention because the odor of flatulence will always be present and feces can leak out of the pouch.Incorrect: Though this is a correct action, the nurse did not identify the actual reason for the client's increased gas production. Another intervention, such as taking an antacid, might be more appropriate. Determine the client's usual eating patterns, including food groups and beverages. Instruct the client to purchase a pouch with a gas filter to allow the excess gas to escape. Suggest that the client puncture a small hole in the top of the pouch to

release the gas. Encourage the client to eliminate carbonated beverages from his diet, as they are gas-producing. 15 A nurse is caring for a client who has acquired immune deficiency syndrome (AIDS). The nurse notes that the client has multiple and widespread raised, purplish-brown skin lesions. Based on the client's immune status and the characteristic appearance of this skin alteration, the nurse suspects that this client has: Incorrect: Molluscum contagiosum is associated with AIDS, and is caused by a virus, but its lesions have a much different appearance. They are usually small, shiny, flesh-colored or pink dome-shaped growths with a small indentation in the center. They easily become red or inflamed. In clients who have AIDS, these lesions are often large and extensive, particularly on the face.Incorrect: Actinic keratosis is not cancerous, but its lesions can evolve into skin cancer. This disorder appears as rough, red, or brown scaly patches on the skin, usually in areas that have been exposed to the sun. It is not specifically associated with AIDS.Correct: Kaposi's sarcoma lesions are malignant integumentary manifestations that are associated with AIDS. These cutaneous lesions are usually brownish-pink to deep purple, and either flat or raised and edematous.Incorrect: Basal cell carcinoma can appear as a small raised bump that has a smooth, pearly appearance, or it can be scar-like and firm to the touch. Prolonged exposure to sunlight's ultraviolet radiation is the most common cause of this type of skin cancer. molluscum contagiosum. actinic keratosis. Kaposi's sarcoma. basal cell carcinoma. 16 A nurse is performing an assessment on a 30-year-old client diagnosed with HIV/AIDS. The client is experiencing mouth pain, difficulty swallowing, retrosternal pain, and unusual gustatory sensations. These symptoms are characteristic of: Incorrect: Cryptococcosis is an opportunistic meningeal infection that is a complication of AIDS. It is caused by Cryptococcus neoformans. Infection with this fungus is characterized by headache, blurred vision, and nuchal rigidity (stiff neck).Incorrect: Mycobacterium avium-intracellulare complex (MAC) is a common opportunistic bacterial infection associated with AIDS. It is caused by Mycobacterium tuberculosis. Infection with this bacterium causes fever, fatigue, weight loss, and lymphadenopathy.Incorrect: Histoplasmosis is an opportunistic fungal infection that is a complication of AIDS. It is caused by Histoplasma capsulatum. Infection with this fungus causes dyspnea, fever, and enlarged spleen, liver, and lymph nodes.Correct: Candida albicans-related stomatitis/esophagitis is an opportunistic fungal infection that is a common complication of AIDS. This infection is characterized by mouth pain, difficulty swallowing, retrosternal pain, and the client's report that food tastes unusual or different. cryptococcosis.

mycobacterium avium-intracellulare complex. histoplasmosis. candidal stomatitis/esophagitis. 17 A nurse is teaching a client at high risk for osteoporosis about dietary measures she can take to increase her calcium level. The nurse advises her to eat plenty of: Incorrect: One medium-size carrot only contains about 14 mg of calcium, making it a poor dietary source of this nutrient.Correct: One cup of cooked spinach contains 200 mg of calcium. Most vegetables contain considerably less calcium, except for bok choy, kale, collard greens with stems, and turnip greens, which are also good sources of calcium.Incorrect: One cup of cabbage only contains about 44 mg of calcium, making it a poor dietary source of this nutrient.Incorrect: One medium-size potato only contains about 14 mg of calcium, making it a poor dietary source of this nutrient. carrots. spinach. cabbage. potatoes. 18 A nurse is evaluating a client's understanding of the risks of hormone replacement therapy (HRT). The client is listing symptoms that would require immediate medical attention if she developed them. Which stated symptom reflects the client's understanding of the risks of this type of therapy? Incorrect: Dysmenorrhea is a common adverse effect of HRT that would not warrant immediate medical attention.Correct: HRT increases the risk of pulmonary embolism due to its thrombogenic effects. This is a life-threatening complication. Dyspnea is the most common symptom of pulmonary embolism; tachycardia is its most common sign.Incorrect: Vaginal candidiasis, an adverse effect of HR, produces a thick, white, cottage-cheese-like discharge. Women who have had this type of infection diagnosed previously are well aware of the symptoms and can obtain over-the-counter fungicides to treat it. Any unusual discharge that does not respond to this treatment should be reported to the provider so that it can be correctly diagnosed and treated.Incorrect: Nausea develops in about 20% of women during the first 2 to 3 months of HRT. Although this can be unpleasant, it is a common adverse effect and does not warrant immediate medical intervention. Dysmenorrhea Tachycardia Vaginal discharge Nausea 19 The most appropriate nursing diagnosis for a client who has hypothyroidism is: Incorrect: Hypothyroidism results in constipation related to gastrointestinal hypomotility.Incorrect: Hypothyroidism results in cold intolerance related to decreased metabolic rate.Incorrect: Hypothyroidism results in weight gain related to decreased

metabolic rate.Correct: Hypothyroidism results in decreased contractility of the heart, resulting in reduced cardiac output that can lead to reduced perfusion of the brain. diarrhea related to gastrointestinal hypermotility. hyperthermia related to heat intolerance secondary to hypermetabolism. altered nutrition: less than body requirements. altered thought processes related to diminished blood flow secondary to decreased cardiac output. 20 A nurse is caring for a client who has just been diagnosed with stage II colorectal cancer. He asks her to tell him more about his condition, how long he's had it, how the doctor knew, and what he can expect. The nurse explains to the client that he: Incorrect: A complete blood count (CBC) can help the provider detect anemia from the effects of the cancer, but it is not a diagnostic indicator. And just like fecal occult blood, anemia can be the result of many other conditions and disorders.Correct: Colorectal cancer is often asymptomatic until it is quite advanced. It is a slow-growing cancer.Incorrect: If at all possible, colorectal surgeons attempt to save the sphincter and avoid a colostomy, so that the client can maintain "normal" bowel function and avoid further disturbances in self-image.Incorrect: The size of the tumor does not necessarily indicate long-term survival. Better prognostic indicators include the degree of penetration of the tumor through the bowel wall, the presence or absence of nodal involvement, and the presence or absence of distant metastases. was diagnosed based on the results of a CBC and fecal occult blood. might have had the condition without symptoms for many years. should expect to need a colostomy as the disease progresses. might think that the size of the tumor predicts his long-term survival. 21 To promote vasodilation and prevent vascular compression in a client with diabetes who has developed peripheral arterial disease, the nurse: Incorrect: Warmth promotes arterial flow by preventing the vasoconstricting effects of cold environmental temperatures. Brisk arterial blood flow is essential for ensuring peripheral tissue perfusion. Improving arterial blow flow also promotes the development of collateral circulation.Correct: Microvascular complications are blood vessel diseases that are especially common and have an earlier onset in clients who have diabetes. Nicotine causes vasospasm, which impedes peripheral circulation significantly.Incorrect: Crossing the legs at any point compresses blood vessels in the legs and thus impedes circulation. Leg crossing, whether at the knees or at the ankles, must be discouraged.Incorrect: Clients who have peripheral arterial disease should be instructed to walk to the point of pain, then rest until the pain eases, then go back to walking. This routine helps build endurance and helps enhance oxygen utilization in the tissues. maintains a cool environment. discourages nicotine use. suggests crossing the legs only at the ankles.

advises limiting walking to avoid pain. 22 A nurse has been monitoring a client's vital signs following the surgical repair of an abdominal aortic aneurysm several days ago. The client's current vital signs are 101 F (38.3 C), pulse 80, respirations 16, and blood pressure 128/76. What should the nurse do next? Incorrect: Although the nurse must notify the provider of this change in the client's status, it would not be the appropriate action for the nurse to take first.Incorrect: Continuing to collect data is always important, however this would not be the appropriate action for the nurse to take first. Also, if the nurse observes a change in status, such as this temperature elevation, it would be appropriate to recheck the client's temperature sooner than the routine interval specifies.Incorrect: Unless fluids are restricted for this client, this would be a helpful intervention for a rise in temperature. However, this would not be the appropriate action for the nurse to take first.Correct: A temperature elevation above 100 F (37.8 C) from the 3rd postoperative day onward is an indication of wound infection. Notify the surgeon of the temperature elevation. Continue to monitor vital signs every 4 hours. Encourage the client to drink more fluids. Assess the surgical incision for signs of infection. 23 A nurse is performing an initial assessment on a client with a history of renal calculi who is again reporting an abrupt onset of severe colicky pain. When the nurse asks the client to identify the location of his pain, the nurse expects him to indicate which area? Incorrect: This is an area of pain more commonly associated with gastroesophageal reflux disease (GERD) or cardiovascular disorders, such as angina pectoris or myocardial infarction. Some clients might experience epigastric discomfort with high calcium levels from hyperparathyroidism, which can lead to the development of renal calculi, but not from the renal calculi specifically.Correct: Clients with renal calculi have classic pain symptoms in either flank. Pain begins in the kidneys, then, as the stones move down the ureter, the pain moves downward through the flank area as the stones move toward the bladder.Incorrect: Pain subsides temporarily while the stones are in the bladder and then resumes as they pass through the urethra.Incorrect: Right mid- to right lower quadrant pain is usually associated with specific gastrointestinal problems, such as constipation, cirrhosis of the liver, and gallbladder disease. As renal calculi progress through the ureters, some clients report abdominal pain, but it would typically be lower in the abdomen. 24 A nurse is planning care for a 68-year-old client with a sensorineural hearing loss who is admitted for an elective surgery. To use a communication style that best meets this client's needs, the nurse must understand that this client is likely to have difficulty: Correct: People who have a sensorineural hearing loss have difficulty distinguishing words. They might have problems hearing consonants, while vowels are clear. This can lead to communication problems unless the nurse works with the client to determine what is most helpful for that particular client, for example, talking into the less-

impaired ear, using gestures, or speaking slowly and distinctly while facing the client so the client can "lip-read."Incorrect: People who have a sensorineural hearing loss are usually able to hear sounds, but they are muffled or distorted.Incorrect: This is more likely in a client who has a conductive hearing loss. These clients tend to speak softly because they perceive their own voice, which is conducted by bone as opposed to the air conduction of others' voices, as excessively loud.Incorrect: People who have sensorineural loss have a diminished ability to hear high-pitched sounds. understanding speech. hearing most sounds. being heard by others. hearing low-pitched sounds. 25 To avoid blood test results that falsely indicate hyperkalemia, the nurse should: Incorrect: A tight tourniquet will increase cell hemolysis, the breakage of the membrane of red blood cells. When this occurs, the cells' components, including potassium, are released into the surrounding fluid. Potassium from hemolyzed cells falsely elevates the specimen's potassium level. It is recommended that blood samples for potassium determination be drawn without a tourniquet or at least be drawn after the tourniquet is released on entry of the needle into the client's vein.Correct: Hemolysis is a breakage of the membrane of red blood cells. When this occurs, the cells' components, including potassium, are released into the surrounding fluid. Potassium from hemolyzed cells falsely elevates the specimen's potassium level. Hemolysis can result from a number of factors, including vigorous shaking of specimens, clotting of specimens, delays in processing, and temperature changes, especially extreme heat or cold. To prevent hemolysis, blood specimens must be handled minimally and delivered promptly to the lab.Incorrect: Intravenous fluids will dilute the blood specimen. And if potassium is being infused in the intravenous solution, a falsely elevated level can result.Incorrect: Exercise can cause muscle cell hemolysis, the breakage of the membrane of red blood cells. When this occurs, the cells' components, including potassium, are released into the surrounding fluid. Potassium from hemolyzed cells falsely elevates the specimen's potassium level. In fact, opening and closing the fist 10 times with a tourniquet in place on the limb actually increases the potassium level in the specimen by 10% to 20%. keep a snug tourniquet on the client's limb while withdrawing the sample. deliver the blood sample to the lab as soon as possible after withdrawing the blood. obtain the blood specimen from a vein just above an intravenous catheter insertion site. have the client open and close the fist repeatedly before the blood is withdrawn. 26 A nurse is caring for a 22-year-old client diagnosed with bacterial meningitis. The nurse's primary rationale for implementing fever-reduction measures for this client is to:

Incorrect: Comfort is an important consideration for any client, however, a client with meningitis usually experiences acute pain from headache. Comfort measures would be focused on pain relief. Fever reduction is also important, but not primarily for promoting this client's comfort.Incorrect: Delirium is a possible manifestation of bacterial meningitis, but it is multifactorial in origin, resulting not just from hyperthermia, but from infection, dehydration, electrolyte imbalances, pain, stress, and other possible pre-existing factors. With fever reduction, there might be some lessening of the client's delirium, but this is not the primary reason for fever-reduction measures for this client.Correct: In a client who has meningitis, fever must be managed aggressively because it increases cerebral edema and the likelihood of seizures. Also, extremely high body temperatures over a prolonged period of time can cause neurologic damage.Incorrect: This is an important consideration because high fever increases metabolic rate and therefore insensible fluid loss, and if this is the case for this client, diaphoresis would further increase those fluid losses. However, this is not the primary reason for treating this client's fever. promote comfort. decrease delirium. reduce cerebral edema. lessen diaphoresis. 27 A client previously diagnosed with myasthenia gravis is brought to an emergency department in myasthenic crisis. Edrophonium chloride (Tensilon) is administered and the client begins to show some improvement. During this initial period of diagnosis and stabilization, the nurse's highest priority, given this client's symptoms, is: Incorrect: Symptoms of myasthenic crisis are similar to those of cholinergic crisis. To differentiate the two, edrophonium chloride (Tensilon) is administered. A client with myasthenic crisis will improve; a client with cholinergic crisis will not improve or will deteriorate. If myasthenic crisis is diagnosed, the client is given neostigmine (Prostigmin), not atropine. If cholinergic crisis is diagnosed, the client is given atropine.Incorrect: Clients who have myasthenia gravis have difficulties swallowing and chewing and are at risk for aspiration due to a decreased gag reflex. Nutritional support is a crucial aspect of managing this chronic disease, however, at the time of crisis and stabilization, this is not the nurse's highest care priority.Incorrect: An important component of teaching a client with myasthenia gravis to manage this chronic disorder is energy conservation, basically scheduling activities to coincide with peak strength levels, pacing activities, and using strategies to reduce unnecessary energy expenditure. However, at the time of crisis and stabilization, client teaching in this area is not the nurse's highest care priority.Correct: Neuromuscular respiratory failure is the most critical complication of myasthenic crisis. Respiration is compromised as weakened respiratory muscles fail to support inhalation. Inadequate cough and gag reflexes result in poor airway clearance. Thus the nurse's highest priority is to keep the client's airway patent and to monitor respiratory status. atropine administration. nutritional support. energy conservation. airway protection. 28

A client with a history of hypertension who has developed stable angina pectoris is prescribed verapamil (Calan). When teaching this client about taking a calcium channel blocker, the nurse will advise the client to avoid taking it with which dietary product? Correct: Grapefruit juice interacts with calcium channel blockers, potentiating their effects. It increases their hypotensive effect and puts the client at risk for drug toxicity. Grapefruit juice raises drug levels by inhibiting intestinal metabolism of the drug, thus making more of the drug available for absorption. This client should take this medication with another beverage, such as water.Incorrect: Orange juice has no known effect on the metabolism of calcium channel blockers, thus this is a safe beverage to take with this client's medication.Incorrect: Apple juice has no known effect on the metabolism of calcium channel blockers, thus this is a safe beverage to take with this client's medication.Incorrect: Tomato juice, unless it is the low-sodium variety, is not a good choice for hypertensive clients due to its relatively high sodium content. However, it has no known effect on the metabolism of calcium channel blockers. Grapefruit juice Orange juice Apple juice Tomato juice 29 A 20-year-old client arrives at an emergency department with a fireworks-related burn injury on the palm of his right hand. The nurse assesses the depth of skin destruction as partial-thickness. Identify the image that depicts a partial-thickness burn in this area. Incorrect: This is an illustration of a superficial burn, formerly called a first-degree burn. Superficial burns, such as sunburn and injuries from quick heat flashes, damage the epidermal layer only, leaving the skin reddened, dry, and painful. After 24 hours, the skin might also blister and peel.Incorrect: This is an illustration of a full-thickness burn, formerly called a third-degree burn. The burned area is blackened or brown in appearance and reaches through the epidermis to the dermis, and possibly the muscles, tendons, and bone. Even though this is the most severe classification of burned tissue, full-thickness burned areas are less painful than partial thickness burned areas because of the nerve destruction involved.Correct: This is an illustration of a partial-thickness burn, formerly called a second-degree burn. This type of injury involves the epidermis and the upper layers of the dermis. The skin appears red and white with blisters, and these burns are characteristically very painful. 30 A nurse is completing an admission assessment on a client who reports moderate diffuse abdominal pain that was gradual in onset months ago, varies in intensity, and is associated with fatigue. Based on this assessment information, the nurse classifies this client's pain as: Incorrect: Neuropathic pain involves abnormal sensation by the peripheral or central nervous system. Examples of this type of pain are phantom limb pain, trigeminal neuralgia, and diabetic neuropathy.Incorrect: Somatic pain arises from bone, joint, muscle, skin, or connective tissue and is usually well localized. Correct: Chronic pain can be gradual or sudden in onset, often waxes and wanes, and can be associated with behavioral manifestations such as fatigue and withdrawal from social

activities.Incorrect: Psychogenic pain is a component of somatization disorder, a syndrome of multiple symptoms that cannot be explained medically and are associated with psychological distress, but the symptoms are neither feigned nor intentionally induced. Pain is usually reported in multiple sites in clients who have this disorder. Based on the nurse's assessment data, there is no reason to believe, without further diagnostic testing, that there is no medical reason for this client's abdominal pain. neuropathic. somatic. chronic. psychogenic. 31 A nurse and a dietician are preparing a class on weight management. An appropriate action for them to discuss with the class participants is: Incorrect: Weekly weight checks are recommended, as weight varies too much on a daily basis. Seeing a sudden increase when the person has adhered carefully to the diet plan can be very discouraging, when in fact it might reflect an alteration in fluid balance or retention of urine or feces.Incorrect: Adequate protein is important for maintaining muscle mass. It is better to avoid rich, calorie-dense foods.Incorrect: If a nutritionally balanced diet plan is carefully planned and followed, vitamin supplements might not be necessary.Correct: Setting realistic goals for weight loss is an important element of success. Trying to lose weight too fast only leads to frustration and defeat. Losing weight quickly also puts people at risk for nutritional deficiencies and inadequate energy. Rapid weight loss is often a loss of body fluid rather than body fat, and as soon as the person deviates from the restrictive diet plan, the weight is quickly regained. checking weight every day at the same time of day. limiting their dietary intake of high-protein foods. taking a multivitamin supplement daily. planning to lose 1 to 2 lb (0.5 to 0.9 kg) per week. 32 When teaching a group of clients about risk factors for acute myocardial infarction, the nurse explains that the presence of which factor reduces their risk for coronary artery disease and subsequent acute MI? Incorrect: Triglyceride levels between 150 and 199 mg/dL are considered borderline high, while levels of 200 to 499 mg/dL are considered high. As triglyceride levels climb above 150 mg/dL, the client's risk for developing coronary artery disease and subsequent acute myocardial infarction escalates.Incorrect: LDL is the "bad cholesterol" that builds up on the inner walls of the coronary arteries and clogs them with plaque. Increased LDL levels raise the risk of coronary artery disease and subsequent acute myocardial infarction. Optimally, LDL should be below 100 mg/dL.Incorrect: Clients whose total cholesterol is between 200 and 239 mg/dL are considered at borderline high levels and are therefore at risk for coronary artery disease and subsequent acute myocardial infarction. Ideally, total cholesterol should be less than 200 mg/dL to reduce a client's risk

of cardiovascular disease.Correct: HDL is the "good cholesterol" that helps remove excess cholesterol from the arteries, thus preventing the buildup of arterial plaque. It is thought that HDL carries cholesterol away from the arteries and back to the liver, from which it is eliminated from the body. HDL levels of 60 mg/dL and above are considered protective against coronary artery disease, thus reducing the risk for acute myocardial infarction. Serum triglycerides above 200 mg/dL Low-density lipoprotein (LDL) above 130 mg/dL Total cholesterol below 240 mg/dL High-density lipoprotein (HDL) above 60 mg/dL 33 Which statement made by a client who has emphysema indicates an understanding of the nurse's teaching about managing this chronic condition? Incorrect: The client should practice abdominal (diaphragmatic) breathing exercises daily. The purpose of these exercises is to avoid exhaustion of the accessory muscles of respiration in the neck and the upper part of the chest. The client focuses instead on using the diaphragm to achieve maximum inhalation and to slow the respiratory rate. By practicing these exercises daily, the client will be able to implement them readily as soon as he experiences dyspnea. Correct: Clients who have emphysema have greater-than-usual nutritional requirements for calories and protein and often need nutritional supplements between meals as well. Weight loss and malnutrition are commonly seen in these clients. Incorrect: Pursed-lip breathing is an effective technique for clients with emphysema to use to prevent bronchiolar collapse and air trapping. However, this technique is performed by first inhaling slowly through the nose, then exhaling slowly through pursed lips. Incorrect: Clients who have emphysema should drink at least 3 liters of fluid per day unless contraindicated for other reasons such as heart failure. However, this fluid should be consumed between meals (not with them) to prevent excess stomach distention, which would exert pressure on the diaphragm. Fluids are essential for clients with emphysema to liquefy secretions and facilitate their expectoration. "I will practice abdominal breathing when I have trouble breathing." "I will follow a daily diet high in calories and protein." "I will inhale slowly through pursed lips to help me breathe better." "I will drink a total of 2 liters of fluid every day with my meals." 34 A 70-year-old client visiting his primary care provider's office asks the nurse if he should get any immunizations. The nurse explains that the Centers for Disease Control and Prevention (CDC) recommends that people of his age get a(n): Incorrect: Pneumococcal disease kills more people in the United States than all other vaccine-preventable diseases combined, and older adults are at especially high risk. The CDC recommends that all people 65 years old and older receive the pneumococcal polysaccharide vaccine. For most people, one dose is sufficient, but if a person received that dose before age 65 and is now 65 or older, another dose is recommended once 5 years have passed since the first dose. But it does not have to be repeated every 2 years.

Correct: Influenza is a significant cause of morbidity and mortality in older adults. In the United States, the CDC recommends that all people 50 years old and above get an influenza immunization each year.Incorrect: Although the fatality rate of tetanus in older adults is especially high, there is no benefit associated with repeating the tetanus-diphtheria booster more often than the currently recommended 10-year interval.Incorrect: The meningococcal vaccine is recommended for people who, regardless of age, have certain medical indications, such as adults who have asplenia, or are traveling to regions where meningococcal disease is epidemic. pneumococcal vaccine every 2 years. influenza vaccine each year. tetanus-diphtheria booster every 5 years. single meningococcal immunization. 35 At a visit with his primary care provider, a client tells the nurse that he feels chest pain every time he walks up the stairs in his home. As soon as he rests, he feels better. This type of chest pain is a typical finding with: Incorrect: Pericarditis, an inflammation of the membranous sac that encloses the heart, causes chest pain that radiates to the left side of the neck, shoulder, or back. The pain is oppressive and gets worse with inspiration, coughing, and swallowing. However, it also worsens when the client lies supine to rest.Correct: Stable angina is chest discomfort that occurs with moderate to prolonged exertion in a pattern that is familiar to the client, and then disappears with rest.Incorrect: The pain of acute myocardial infarction typically lasts 30 minutes or more and is unrelieved by any precipitating factors that might have triggered it.Incorrect: The pain of pulmonary embolism is pleuritic in nature; it is sharp and stabbing but increases with inspiration and is unrelieved by rest. Although it is considered a respiratory condition, it does originate from some type of particulate matter that enters the cardiovascular system and obstructs a pulmonary vessel. pericarditis. stable angina. acute myocardial infarction. pulmonary embolism. 36 A nurse is caring for a postoperative client who had a kidney transplantation. Because it is a sign of life-threatening complications of this surgery, it is the nurse's assessment priority to monitor this client for: Incorrect: After renal transplantation, the urine is initially bloody or pink and does not usually return to normal for several days or weeks.Incorrect: It is quite common for very large amounts of urine to be produced soon after the transplanted kidney's blood supply is reestablished, possibly as much as 1 liter/hour. This is not a sign of complications, but requires careful monitoring and fluid replacement to prevent dehydration and consequent renal hypoperfusion and renal tubular damage.Incorrect:

Although the catheter must be cleared promptly so that pressure in the bladder does not build and put the graft site at risk, this is not in itself a life-threatening complication. The usual cause of an obstructed catheter after renal transplant is blood clots. Continuous bladder irrigation is often prescribed to prevent this.Correct: An abrupt decrease in urinary output can reflect the onset of rejection of the transplanted organ, acute tubular necrosis, or thrombosis. excretion of bloody urine. rapid, large-volume diuresis. obstruction of the indwelling catheter. a sudden drop in urine output. 37 A 37-year-old client is admitted to an emergency department after sustaining deep partialthickness and full-thickness burns over 36% of his body in a residential fire. After the client's airway is stabilized, fluid requirements for the first 24 hours are calculated using the Parkland formula. The client weighs 165 lb (75 kg). Indicate (in numerals with no punctuation) the amount in milliliters of lactated Ringer's this client will receive intravenously during the first 8 hours. mL Correct: The Parkland formula for fluid replacement for the first 24 hours after a burn injury is calculated by multiplying 4 milliliters times the number of kilograms of body weight times the percentage of total body surface area (TBSA) burned. On the first day, half of that total is to be infused over the first 8 hours, with the other half infused over the next 16 hours. So, for this client, the equation would be: 4 mL X 75 kg X 36 = 10,800. Half of that amount will be infused over the first 8 hours, so 10,800/2 = 5,400 mL. Without the comma, the correct answer is: 5400. Any calculation that yields a response other than 5400 would be incorrect.Incorrect: The Parkland formula for fluid replacement for the first 24 hours after a burn injury is calculated by multiplying 4 mL times the number of kilograms of body weight times the percentage of total body surface area (TBSA) burned. On the first day, half of that total is to be infused over the first 8 hours, with the other half infused over the next 16 hours. So, for this client, the equation would be: 4 mL X 75 kg X 36 = 10,800. Half of that amount will be infused over the first 8 hours, so 10,800/2 = 5,400 mL. Without the comma, the correct answer is: 5400. Any calculation that yields a response other than 5400 would be incorrect. 38 A nurse is assessing a 74-year-old client diagnosed with left-sided heart failure. Which would the nurse expect this client to have? Incorrect: Dependent edema is a classic manifestation of right-side heart failure. Pressing edematous skin with a finger leaves a transient indentation called pitting edema.Incorrect: The jugular venous system reflects the hemodynamics of the right side of the heart. Right-sided heart failure raises the pressure and volume within the jugular veins, making them visibly distended when the client is sitting or the head of the bed is elevated more than 30 degrees.Correct: Orthopnea, or shortness of breath when the person is in a recumbent position, is a characteristic manifestation of left-sided heart failure. It is caused by increased lung pressures from interstitial and alveolar edema.Incorrect: This is a classic finding with right-sided heart failure, as venous engorgement causes hepatomegaly

resulting in tenderness over the liver. Pitting edema of the lower legs Jugular venous distention Shortness of breath while lying down Right upper quadrant pain 39 While changing a client's dressing, the nurse observes that the abdominal incision has dehisced, leaving a large amount of brown drainage on the surgical dressing. What should the nurse do first? Incorrect: The nurse will of course notify the surgeon, but must provide immediate client care and assessment prior to leaving the client's bedside. If the nurse can alert another staff member to notify the surgeon while she provides client assessment and care, that would be an appropriate intervention.Incorrect: Continued assessment is always an important nursing function. However, this would not be the appropriate action for the nurse to take first. Unless the wound is actively hemorrhaging or there are other reasons for this client to be hypovolemic, the nurse would have no reason at this time to suspect that the client is in shock.Correct: The wound bed must be kept moist until the surgeon can inspect the wound and prescribe treatment.Incorrect: The nurse can give the client emotional support during all nursing actions by conveying a caring, compassionate, and respectful approach. However, specific, directed interventions for providing emotional support are not the highest care priority for this client at this particular time. Notify the surgeon of the dehiscence immediately. Monitor the client's vital signs for indications of shock. Cover the wound with a sterile, saline-moistened dressing. Provide the client with emotional support interventions. 40 A client diagnosed with Crohn's disease is prescribed sulfasalazine (Azulfidine). In teaching the client about taking this medication, the nurse instructs her to: Incorrect: Sulfasalazine can cause a yellow-orange tint noticeable in the urine and on the skin. It can also discolor contact lenses.Incorrect: Sulfasalazine should be taken after meals to prevent gastrointestinal discomfort.Correct: Sulfasalazine interacts with anticoagulants to potentiate their anticoagulant effect. Due to the increased risk of bleeding tendencies associated with aspirin use, clients taking sulfasalazine should avoid taking aspirin. Both drugs can increase the risk for gastrointestinal distress. Sulfasalazine can also cause blood dyscrasias.Incorrect: Sulfasalazine should be taken with a full glass of water to help the client maintain adequate hydration. It is important for the client to increase fluids to 2 L/day to reduce crystallization in the kidneys. expect that her urine and skin might darken. take the medication prior to meals if possible. avoid taking aspirin with this medication. take the drug with minimal water for maximal effect. 41

Which intervention is the nurse's highest priority in managing an acute asthma episode? Correct: Beta-adrenergic medications decrease the inflammatory response that triggers narrowing of the airways. They provide prompt relief of airflow obstruction and are the first priority when a client presents with an acute asthma exacerbation.Incorrect: Although not the highest priority when a client presents with an acute asthma exacerbation, it is an important measure for managing the client's dyspnea, and in fact, the client might immediately assume this position on arrival for care. Postioning the client in high-Fowler's, leaning forward with the arms propped on an overbed table, promotes chest expansion and optimal gas exchange.Incorrect: Adequate rest is essential for the client's recovery from the asthma exacerbation, but it will not relieve the client's dyspnea and wheezing.Incorrect: Assessment of the client is always essential, but many clients with asthma have a nonproductive cough. In any case, an acute asthma exacerbation requires immediate relief of the client's dyspnea. Administering nebulized beta-adrenergics as prescribed Positioning the client in high-Fowler's Providing immediate rest for the client Assessing sputum amount, color, and consistency 42 A nurse is reviewing the history of a client who is experiencing signs and symptoms of acute glomerulonephritis (GN). The nurse notes that the client has previously been diagnosed with a disease that increases the risk for developing acute GN. Which is a secondary glomerular disease that increases this risk? Correct: Systemic lupus erythematosus (SLE) is a secondary glomerular disease that puts clients at risk for acute GN, because it adversely affects renal function and can cause glomerular damage.Incorrect: Rheumatoid arthritis is not a cause of acute GN, and it does not result in glomerular damage.Incorrect: Prostate cancer is not a cause of acute GN, and it does not result in glomerular damage.Incorrect: Diabetes insipidus is not a cause of acute GN, and it does not result in glomerular damage. Systemic lupus erythematosus Rheumatoid arthritis Prostate cancer Diabetes insipidus 43 A client who has osteoarthritis has undergone arthroscopy of the left knee. After the procedure, the nurse is teaching the client to walk with crutches using a 4-point gait. Prioritize the order in which the client will perform the steps of the 4-point gait after assuming the initial tripod position, indicating in the correct order the 4 numbers corresponding to the steps, with no spaces or punctuation between them. 1. Advance right foot. 2. Advance right crutch. 3. Advance left crutch. 4. Advance left foot.

Correct: The 4-point gait is a simple and safe crutch-walking gait, providing at least three points of support. Some coordination is required, however, so it is important for the nurse to make sure the client is comfortable with the steps and is performing them accurately and in the correct sequence prior to discharge. The proper standing position, called the tripod stance, involves standing with the feet slightly apart, with the crutches placed about 6 inches (15 cm) in front of the feet and out laterally about 6 inches (15 cm). This creates a wide base of support. Then, for the 4-point gait, the client moves the right crutch forward about 4 to 6 inches (10 to 15 cm), then moves the left foot forward to the level of the left crutch. Then the client moves the left crutch forward, followed by the right foot. Any other sequence for these steps would be incorrect.Incorrect: The 4-point gait is a simple and safe crutch-walking gait, providing at least three points of support. Some coordination is required, however, so it is important for the nurse to make sure the client is comfortable with the steps and is performing them accurately and in the correct sequence prior to discharge. The proper standing position, called the tripod stance, involves standing with the feet slightly apart, with the crutches placed about 6 inches (15 cm) in front of the feet and out laterally about 6 inches (15 cm). This creates a wide base of support. Then, for the 4-point gait, the client moves the right crutch forward about 4 to 6 inches (10 to 15 cm), then moves the left foot forward to the level of the left crutch. Then the client moves the left crutch forward, followed by the right foot. Any other sequence for these steps would be incorrect. 44 At highest risk for cervical cancer is a client who: Incorrect: Age at menarche has not been associated with developing cervical cancer.Incorrect: Oral contraceptive use slightly increases the risk of cervical cancer but only in women who have certain other definitive risk factors for cervical cancer.Incorrect: Having sexual intercourse prior to age 20 increases the risk of cervical cancer.Correct: Some subtypes of human papilloma virus (HPV) have been associated with cervical cancer, thus HPV infection is considered a risk factor for developing cervical cancer. began menstruating at age 10. took an oral contraceptive for 2 years. first had sexual intercourse at age 30. has been diagnosed with human papilloma virus. 45 A client who is scheduled to undergo a total prostatectomy for treatment of prostate cancer asks the nurse what he can expect in terms of sexual function after the surgery. The nurse explains that: Incorrect: With some types of prostate surgery, this is appropriate information, as that is how long it typically takes for the prostatic fossa to heal.Incorrect: It is not for the nurse to decide what the client is concerned about at any point during the course of his cancer treatment. The nurse should address the client's concerns, explaining what he can realistically expect in the area of sexual function after the surgery.

Correct: Although the nurse must confirm what the client is probably quite apprehensive about, the client also needs to understand that there are options available that he can discuss with his physician. These options can help him continue to have sexual intercourse, and include penile implants, negative pressure devices, and pharmacological therapies. Incorrect: Sexual dysfunction is a complication after prostate surgery due to potential damage to the pudendal nerves. The nurse should explain the risks associated with his particular type of surgery and whether he can expect any sexual dysfunction to be temporary or permanent. he can resume his usual sexual activities in 6 to 8 weeks. he should focus first on being cancer-free after treatment. impotence almost always follows total prostatectomy. impotence can occur immediately after surgery, but it is rare. 46 A nurse is preparing a client for discharge after treatment for tonic-clonic seizures. The nurse instructs the client and family to take which action immediately if the client has a seizure at home? Incorrect: Nothing should be placed in the person's mouth during a seizure because of the risk of injury to the mouth and teeth while the person is clamping down on the object. The family member might also be injured while attempting to force an object between the person's clenched teeth.Correct: The first priority in caring for a person having a seizure is to keep the airway patent. Constricting clothing can restrict breathing and circulation, so the family should loosen any tight clothing right away.Incorrect: When a client has received or is receiving treatment for tonic-clonic seizures, it is not necessary to call emergency services (paramedics) for a single seizure unless there are complicating factors such as potentially serious injuries. Emergency services should only be called for life-threatening emergencies or situations that can become life-threatening if the client is moved or transported by untrained individuals.Incorrect: The person should be placed in a side-lying position to prevent aspiration. Insert a tongue protector. Loosen constricting clothing. Call 911 (emergency services). Place the person in the supine position. 47 A nurse reviews self-care techniques with a client who has moderate, erosive rheumatoid arthritis. She reinforces the concept of joint protection when she reminds the client to: Correct: Joint protection is a means of modifying tasks to reduce the stress placed on joints during routine activities. It involves work simplification techniques as well as planning rest and energy conversation periods between tasks. The finger and hand joints have more pressure exerted on them when wringing a sponge than when pressing on it to remove excess water, so this would be an appropriate joint protection technique.Incorrect: Lifting places more stress on the joints than sliding or pushing a heavy object does. This action would not be considered a joint protection technique.Incorrect: Although this might

allow rest during the other days of the week, it would be too exhausting and place far too much stress on the joints over a brief period of time. An essential principle of joint protection is energy conservation. Household tasks should therefore be spread throughout the week rather than attempted all at one time.Incorrect: Repetitious tasks should be avoided, especially for long periods of time. The client shouldn't knit for long periods of time and should rest between vacuuming rooms, for example. press water from a sponge rather than wringing it. lift objects instead of sliding or pushing them. finishing weekly household tasks within 1 or 2 days. engage in repetitive tasks that keep joints mobile. 48 A nurse expects a client with a diagnosis of acute respiratory failure due to exacerbation of chronic obstructive pulmonary disease (COPD) to have which arterial blood gas result? Incorrect: Clients who have a COPD exacerbation tend to have an elevated PCO2 that is compensated by high HCO3- levels. With acute respiratory failure, however, the body is no longer able to compensate for the elevated PCO2.Correct: Clients who have a COPD exacerbation usually have an elevated PCO2 that is compensated by high HCO3levels, resulting in a normal pH. With respiratory failure, however, the kidneys are no longer able to compensate for the elevated PCO2, and acidosis (a pH below 7.35) results.Incorrect: Clients who have a COPD exacerbation tend to develop hypercapnic respiratory failure compensated by high HCO3- levels. With this type of acute ventilatory failure, the HCO3- remains elevated, but not to a level that will completely compensate for the abnormal PCO2 level.Incorrect: Clients who have a COPD exacerbation use a hypoxic drive for breathing; therefore the PO2 will be below normal. PCO2 below 35 pH below 7.35 HCO3- below 22 PO2 above 90 49 A nurse is caring for a new postoperative client who had a transurethral resection of the prostate (TURP). His vital signs are stable. Continuous bladder irrigation has been started prior to the client's arrival on the medical-surgical unit and the urinary output through the indwelling urinary catheter is dark cherry red with a few noticeable clots. What would be the priority nursing intervention on receiving this client on the unit? Incorrect: With a TURP, the surgical procedure is performed directly through the urethra to the prostate. One of the benefits of this procedure is that the client has no surgical incision. Correct: If the output through the Foley catheter is dark cherry red with clots, irrigating the catheter gently with 30 to 50 mL of bladder irrigating solution can help remove any clots. If continuous bladder irrigation (CBI) is set up, the goal is to have the output pink in color with no visible clots. Thus the flow rate of the CBI should be increased to achieve the desired result.Incorrect: Although the client will need to ambulate, this is not a priority on arrival to the unit from postanesthesia recovery. In the postoperative period, clients who undergo prostate surgery should avoid walking for extended periods of time because it can

increase abdominal pressure and thus increase bleeding and clot formation.Incorrect: Some incontinence or dribbling might occur after prostate surgery. However, teaching clients what to expect after recovery should be done preoperatively and reinforced prior to discharge, along with strategies for managing this problem. But this is not an appropriate intervention in the immediate postoperative period. Checking the incision site for bleeding Gently irrigating the indwelling urinary catheter Ambulating the client to help mobilize clots Telling the client to expect some urinary incontinence 50 A 55-year-old client admitted with intermittent epigastric pain is scheduled for an esophagogastroduodenoscopy (EGD). The nurse provides appropriate pre-procedure education by saying: Incorrect: This describes a barium swallow examination of the gastrointestinal tract, usually done to diagnose duodenal ulcers.Correct: EGD, the most accurate way to diagnose peptic ulcer disease, allows for direct visualization of the ulcerative sites. The provider can also take specimens for biopsy if indicated.Incorrect: This describes a urea breath test done to detect Helicobacter pylori, a spiral-shaped bacterium found in the stomach. H. pylori is associated with the development of most peptic and duodenal ulcers. These bacteria weaken the stomach's defenses by thinning the mucous coating of the stomach, making it more susceptible to the damaging effects of acid and pepsin. Both the bacterial infection and the acid irritate the lining of the stomach, causing an ulcer.Incorrect: EGD is a diagnostic procedure, not a treatment or cure for epigastric pain related to peptic ulcer disease. "You will drink a chalky solution to coat your stomach lining and then we'll take x-rays." "This procedure lets the doctor look inside your stomach and see if there are any ulcers there." "You will have to drink a carbon-enriched urea solution and inhale into a collection container." "This procedure will allow the doctor to eliminate your pain at its source, so it won't recur."

51 A nurse is assessing the effectiveness of the stump dressing on the leg of a client who had a left below-the-knee amputation. The nurse anticipates a possible complication and the need for re-dressing the stump when he finds: Correct: If the dressing is applied correctly, it will reduce or prevent edema and allow for better prosthesis fitting.Incorrect: A tourniquet is only applied for life-threatening bleeding. It is not used routinely on a stump.Incorrect: This is the outcome the nurse is looking for when applying a stump dressing. It helps prepare the limb for better fitting of the prosthesis.Incorrect: The compression of the stump dressing helps reduce pain and

therefore is not a sign of impaired sensation, which would alert the nurse to a possible complication. 3+ pitting edema around the stump dressing. that the tourniquet does not stay on securely. the dressing forming a cone shape over the stump. that the client's reported pain levels have decreased. 52 A 75-year-old client with a long history of peripheral arterial disease is on his way to the medical-surgical unit from postanesthesia recovery 2 hours postoperatively after femoralpopliteal bypass grafting. His vital signs are reported to be stable, with no excessive bleeding noted at the incisional site and palpable distal pulses. Which nursing intervention is the receiving nurse's highest priority when this client arrives on the unit? Incorrect: A client 2 hours postoperatively after peripheral arterial revascularization surgery should be kept on bedrest with the operative leg flat, with no flexion of the knee or hip, at least for the rest of the postoperative day.Incorrect: This is an important nursing intervention for this client. However, when clients are transferred from postanesthesia recovery, pain is usually controlled when they arrive on the medical-surgical unit. Typically, minimal pain is one of the criteria for discharge from a postanesthesia care unit.Correct: By checking pulses, capillary refill, extremity pallor or cyanosis, numbness or tingling in the extremities, and any coldness in the extremity as compared with the nonoperative leg, the nurse is evaluating the arterial flow achieved with revascularization. Any of these signs or changes in the client's condition could indicate an occlusion of the bypass graft, which would have to be reported immediately to the surgeon. This is the highest nursing priority for this client at this time, especially because postoperative graft occlusion often occurs within the first 24 hours of surgery.Incorrect: It is important for the nurse to teach the client how to thoroughly inspect his legs and feet thoroughly and daily and what signs and symptoms to look for and report to his health care provider if found. The client will certainly need this information prior to discharge, but he is unlikely to retain any of it if presented to him 2 hours postoperatively. Ambulate the client to increase blood flow and return through the new graft. Assess the severity of the client's pain using a standardized pain scale. Assess the client's circulation in the operative leg as compared with the nonoperative leg. Teach the client how to inspect his legs and feet and report significant findings immediately. 53 A client who has been diagnosed with a pulmonary embolism is becoming hypoxic and is hyperventilating. As a result, she is developing respiratory alkalosis without compensation. Which arterial blood gas results would the nurse expect to see? (Check all that apply.) Correct: pH 7.50 is a correct response. Whenever there is alkalosis, the pH, normally 7.35 to 7.45, is elevated. PO2 89 mm Hg is an incorrect response. The client is hypoxic, therefore PO2, normally 80 to 100 mm Hg, would be low. PCO2 29 mm Hg is a correct response. With respiratory alkalosis and increased ventilation, PCO2 drops below the

normal range of 35 to 45 mm Hg. O2 saturation 84% is a correct response. The client is hypoxic, therefore oxygen saturation, normally between 96% and 100%, would be low. PCO2 48 mm Hg is an incorrect response. With respiratory alkalosis and increased ventilation, PCO2 drops below the normal range of 35 to 45 mm Hg. HCO3- 28 mEq/L is a correct response. With respiratory alkalosis, bicarbonate remains within the normal range of 22 to 26 mEq/L if the acid-base imbalance is uncompensated. Bicarbonate would drop if the kidneys were to compensate by excreting bicarbonate. Incorrect: pH 7.50 is a correct response. Whenever there is alkalosis, the pH, normally 7.35 to 7.45, is elevated. PO2 89 mm Hg is an incorrect response. The client is hypoxic, therefore PO2, normally 80 to 100 mm Hg, would be low. PCO2 29 mm Hg is a correct response. With respiratory alkalosis and increased ventilation, PCO2 drops below the normal range of 35 to 45 mm Hg. O2 saturation 84% is a correct response. The client is hypoxic, therefore oxygen saturation, normally between 96% and 100%, would be low. PCO2 48 mm Hg is an incorrect response. With respiratory alkalosis and increased ventilation, PCO2 drops below the normal range of 35 to 45 mm Hg. HCO3- 28 mEq/L is a correct response. With respiratory alkalosis, bicarbonate remains within the normal range of 22 to 26 mEq/L if the acid-base imbalance is uncompensated. Bicarbonate would drop if the kidneys were to compensate by excreting bicarbonate. pH 7.50 PO2 89 mm Hg PCO2 29 mm Hg O2 saturation 84% PCO2 48 mm Hg HCO3- 28 mEq/L 54 A nurse in an endocrinology practice has completed an assessment of four clients who have hyperthyroidism. The client most likely to be given priority for thyroidectomy is the client who: Incorrect: A client in thyrotoxic crisis is extremely unstable. The priority for this client is aggressive medical management, possibly intensive care, before surgery can be considered.Incorrect: A client should have well controlled blood glucose levels prior to undergoing a thyroidectomy. Correct: To ensure the best possible surgical outcome for clients who have an appropriate indication for thyroidectomy, clients preparing for thyroid surgery are given antithyroid drugs and iodine to induce a euthyroid state. Surgery is usually reserved for hyperthyroid clients who have not responded to other forms of treatment, have a goiter that compresses the trachea, or have a possible malignancy.Incorrect: Exophthalmos is a classic finding with Graves' disease, one type of hyperthyroidism. It is a protrusion of the eyeballs resulting from impaired drainage from the orbits. It can cause serious complications like corneal ulcers and vision loss, however, the method of treatment of the client's condition

would not be influenced by the presence of this sign of the disease. For most nonpregnant adults, the treatment of choice is radioactive iodine therapy. Nevertheless, exophthalmos does not always resolve with treatment, whether surgical, radiological, or medical. is in thyrotoxic crisis. has blood glucose levels above 200 mg/dL. is presently euthyroid. has bilateral exophthalmos. 55 A nurse is preparing a client with supraventricular tachycardia for elective cardioversion. Which type of medication, if the client is receiving it, should be withheld for 48 hours prior to the cardioversion procedure? Incorrect: Anticoagulants can actually be beneficial during cardioversion because they might help prevent the formation of blood clots that could be released into the client's circulation.Incorrect: Antiarrhythmic agents are beneficial because they help reduce the client's risk for dysrhythmias following cardioversion.Incorrect: Sedatives are generally administered to clients prior to cardioversion to reduce their anxiety and to minimize the discomfort associated with the procedure.Correct: Cardiac glycosides such as digoxin (Lanoxin) are withheld prior to cardioversion. These drugs can increase ventricular irritability and can put the client at risk for ventricular fibrillation after the synchronized countershock of cardioversion. Anticoagulant Antiarrhythmic Sedative Cardiac glycoside A nurse is caring for a client on postoperative day 1 after a left-hip open reduction with internal fixation. It is 0830 and physical therapy is scheduled to work with the client at 0900. What is the highest priority nursing intervention for this client at this time? Incorrect: Although the client might appreciate feeling clean and refreshed in preparation for physical therapy, bathing and dressing activities can potentially tire him. It is more important to help the client conserve energy beforehand to help ensure an effective therapy session.Incorrect: Although this is information he will need, this client is 1 day postoperative. This is probably his first physical therapy session since the surgery. The therapist will go over all this information with the client during the session, thus it is not the nurse's priority to teach him this information at this time.Incorrect: Part of the physical therapist's assessment and goals for a client who has had hip surgery is to plan to teach him the proper techniques for getting in and out of bed and help him achieve total independence in this activity. It is not the nurse's priority to assist the client into a wheelchair 30 minutes before his scheduled therapy session.Correct: Without adequate pain medication to relieve the client's pain and enough time to ensure an optimal analgesic effect, the client's potential to work with the physical therapist might be impaired and the session ineffective. Complete all morning care for the client, including bathing and providing a clean gown. Teach the client which positions to avoid when moving around during the therapy session.

Assist the client out of bed into the wheelchair so he will be ready for physical therapy. Assess the client's pain level, administering adequate and appropriate pain medication. 57 A client is brought to the post anesthesia care unit (PACU) after a diagnostic breast biopsy with node dissection. In the PACU, which would be a priority nursing diagnosis for this client? Incorrect: Appropriate wound care is a priority after surgery, however, at this stage of recovery, the nurse would only focus on the wound if there were signs of hemorrhage or dehiscence.Correct: Immediately after anesthesia administration, the priority is for the client to regain control of the respiratory center and maintain effective respiratory function spontaneously.Incorrect: This might become a higher priority after the client has recovered from anesthesia, but it would not be the nurse's priority at this time.Incorrect: So soon after surgery, the wound would not show signs of infection, nor would the client show signs of systemic infection, such as from invasive catheters or immobility. Impaired skin integrity Ineffective breathing pattern Body image disturbance Potential for infection 58 An immunocompromised adult client planning a trip to Central America asks the nurse in his primary care provider's office how he can protect himself from contracting hepatitis A. After reviewing the modes of transmission of hepatitis A with him, the nurse validates that he understands what to do when he says: Incorrect: The hepatitis A vaccine is recommended for people traveling to areas where they are at increased risk for contracting hepatitis A.Incorrect: The client does not understand that the water used to wash the fruits and vegetables might contain fecal or other contaminants that could transmit the hepatitis A virus.Incorrect: Gloves are no guarantee that the fresh greens and fruit haven't been contaminated by unsafe water or unhygienic practices while a worker is wearing gloves. In addition, the food could have been contaminated during harvesting or processing before it reached the restaurant. Heating food to 185 F (85 C) for 1 minute effectively inactivates the hepatitis A virus, so the nurse should advise this client that it is safer to eat cooked produce.Correct: Removing the outer portion of produce such as bananas will reduce the risk of transmitting the virus. However, it is important to remind this client that, if he handles the outside of the banana with his bare hands, does not wash them, and then handles the inside portion he will eat, he will still be at risk. Peeling the banana from the top, then eating the fruit without touching the portion he eats, then discarding the peel and washing his hands thoroughly would reduce his risk of ingesting the virus. "As long as I follow food and water precautions, I don't need an immunization." "I will only eat fresh produce that has been washed thoroughly." "I can eat salad and fresh fruit in a restaurant where the workers wear

gloves." "I will eat fresh fruits that I can peel myself, such as bananas." 59 Which laboratory result, in addition to an elevated serum sodium level, best indicates that a client with a fluid volume loss has hypernatremia? Incorrect: This value increases with carbon monoxide poisoning, which affects gas exchange and ultimately neurological status, not fluid balance (at least not directly).Incorrect: This value increases in clients who have hypernatremia. Measured to evaluate renal function, blood urea nitrogen (BUN) is a measure of the serum level of the nitrogen portion of urea, the end product of protein metabolism. Dehydration causes BUN levels to rise.Correct: Serum osmolality greater than 300 mOsm/kg H2O reflects hypernatremia. Serum osmolality increases because solutes are leaving the cells and concentrating in the circulation.Incorrect: With dehydration and consequent hypernatremia, it is typical for hematocrit levels to rise. Increased carboxyhemoglobin Decreased blood urea nitrogen Increased serum osmolality Decreased hematocrit 60 A nurse is making initial rounds on an oncology unit. As the nurse prioritizes how she will deliver care for the clients she is assigned, she determines that the client who needs her most immediate intervention is the one who has: Incorrect: Sores in mucous membranes, called mucositis, and specifically in the mouth, called stomatitis, are common in clients undergoing chemotherapy. This client will need local relief from the pain of these ulcers prior to meals, but this condition does not warrant high-priority nursing intervention at this time.Incorrect: The nurse would encourage the client to drink more fluids, but the client would not need priority nursing intervention at this time.Correct: Sudden bleeding from orifices is a sign of disseminated intravascular coagulation (DIC), a life-threatening oncologic emergency that indicates a problem with blood clotting. In oncology, DIC is most often associated with leukemia and some cancers of the lung, pancreas, stomach, and prostate.Incorrect: Nausea and vomiting are common adverse effects of chemotherapy. Although uncomfortable for the client, these effects do not warrant high-priority nursing intervention. multiple painful mouth ulcers. voided 200 mL of dark amber urine. a sudden onset of nosebleeds. ongoing nausea and vomiting. 61 A 50-year old client is admitted with a diagnosis of acute cholecystitis. She reports severe pain and spasms and has fever, persistent vomiting, and jaundice. When planning care for this client, the nurse will include (check all that apply):

Correct: Maintaining gastric decompression is a correct response. When a client with cholecystitis has severe nausea and vomiting, gastric decompression via nasogastric tube is often prescribed to prevent any further stimulation of the gallbladder and to relieve nausea and vomiting. Positioning the client in high Fowler's is an incorrect response. This position increases pressure on the right upper quadrant, the usual primary location for gallbladder-related pain. Low Fowler's is the position of choice for this client. Increasing fiber in the diet as tolerated is an incorrect response. At this point the client might not be tolerating any food at all, but as she recovers from the acute episode, fatty and fibrous foods should be discouraged, as they can cause nausea and spasms. Administering antibiotics as prescribed is a correct response. Antibiotics are often prescribed for clients who have cholecystitis to control any possible infection. Providing nonpharmacologic pain-relief measures is a correct response. This client is in severe pain and her pain might not be immediately or fully controlled with the prescribed analgesics. Nonpharmacological, complementary pain-relief interventions such as guided imagery can help reduce pain and enhance the effects of analgesic agents. Administering anticholinergic agents as prescribed is a correct response. Antispasmodic drugs such as anticholinergics are often prescribed for clients who have cholecystitis to relax the smooth muscles, thus preventing biliary contraction, minimizing secretions, and relieving pain.Incorrect: Maintaining gastric decompression is a correct response. When a client with cholecystitis has severe nausea and vomiting, gastric decompression via nasogastric tube is often prescribed to prevent any further stimulation of the gallbladder and to relieve nausea and vomiting. Positioning the client in high Fowler's is an incorrect response. This position increases pressure on the right upper quadrant, the usual primary location for gallbladderrelated pain. Low Fowler's is the position of choice for this client. Increasing fiber in the diet as tolerated is an incorrect response. At this point the client might not be tolerating any food at all, but as she recovers from the acute episode, fatty and fibrous foods should be discouraged, as they can cause nausea and spasms. Administering antibiotics as prescribed is a correct response. Antibiotics are often prescribed for clients who have cholecystitis to control any possible infection. Providing nonpharmacologic pain-relief measures is a correct response. This client is in severe pain and her pain might not be immediately or fully controlled with the prescribed analgesics. Nonpharmacological, complementary painrelief interventions such as guided imagery can help reduce pain and enhance the effects of analgesic agents. Administering anticholinergic agents as prescribed is a correct response. Antispasmodic drugs such as anticholinergics are often prescribed for clients who have cholecystitis to relax the smooth muscles, thus preventing biliary contraction, minimizing secretions, and relieving pain. maintaining gastric decompression. positioning the client in high Fowlers. increasing fiber in the diet as tolerated. administering antibiotics as prescribed. providing nonpharmacologic pain-relief measures. administering anticholinergic agents as prescribed. 62 A 60-year-old client comes to an emergency department reporting a sudden onset of irregular palpitations, fatigue, and dizziness. The nurse takes the client's vital signs,

auscultating the apical pulse and palpating the radial pulse. Both are rapid and irregular and there is a significant pulse deficit. As the nurse obtains a 12-lead electrocardiogram, she anticipates, based on her assessment data, that this client has: Correct: Atrial fibrillation causes a disorganized twitching of the atrial muscles with a consequent reduction in cardiac output, manifested in some clients as fatigue and dizziness. Conduction through the atrioventricular node occurs randomly, accounting for irregular pulsation. In clients who have atrial fibrillation, the apical pulse is typically faster than the radial pulse. This is because of the failure of some of the client's ventricular contractions to produce peripheral pulse waves strong enough to detect by palpation.Incorrect: Sinus bradycardia is a slow heart rate with a regular rhythm. It can certainly produce symptoms of fatigue and dizziness, but the client's pulses would be regular.Incorrect: First-degree atrioventricular block is a conduction delay, but the sinus impulses do eventually reach the ventricles. It causes no symptoms unless the underlying rhythm is abnormal, in which case any symptoms the client has would be related to the underlying dysrhythmia and not to the first-degree heart block.Incorrect: Ventricular tachycardia is considered an emergency because many clients who develop this dysrhythmia become unresponsive and pulseless. But the atrial and ventricular rhythms are usually regular. atrial fibrillation. sinus bradycardia. 1st-degree atrioventricular block. ventricular tachycardia. 63 A 34-year-old client is brought to an emergency department after sustaining a head injury in a motor-vehicle crash. During the initial assessment for signs of skull fracture, the nurse observes Battle's sign, which is: Incorrect: A linear fracture of the base of the skull is associated with rhinorrhea cerebrospinal fluid leaking from the nose - because this type of fracture generally crosses a sinus and tears the dura. However, this is not called Battle's sign.Correct: Battle's sign is an area of bruising, usually oval-shaped, seen over the mastoid - behind the client's ear. This is associated with a basal skull fracture, along with other common manifestations including facial paralysis and deviated gaze.Incorrect: Bulging of the tympanic membrane caused by a buildup of blood or cerebrospinal fluid is associated with parietal and basal skull fractures. However, this is not called Battle's sign.Incorrect: Called "raccoon's eyes," ecchymosis around the eyes is associated with an orbital skull fracture. However, this is not called Battle's sign. cerebrospinal fluid leakage from the nose. ecchymosis behind the client's ear. bulging of the tympanic membrane. periorbital ecchymosis. 64 A 75-year-old client is experiencing chronic constipation, characterized by infrequent, hard, dry stools. To help correct his constipation, the nurse advises this older client to use which intervention until his constipation is relieved?

Incorrect: A bulk-producing agent such as psyllium hydrophilic mucilloid (Metamucil) is not the treatment of choice for an older adult. These agents can cause fecal obstruction and impaction.Correct: Fecal stool softeners such as docusate sodium (Colace) promote moist, bulky stools that the client can eliminate more easily. Stool softeners are considered safe and have few side effects. Older clients might have difficulty controlling constipation by exercising (due to limited mobility) and eating high-fiber foods (because of dental problems).Incorrect: It is important to discourage older clients from taking laxatives on a regular basis and developing a laxative dependence, the inability to have a bowel movement without taking laxatives. Saline cathartics such as milk of magnesia (MOM) should not be taken more than once a week.Incorrect: Older clients should not selfadminister Fleet's enemas to correct chronic constipation. Enemas can cause tissue trauma in the rectal area, fluid and electrolyte imbalances, vagal nerve stimulation, and dependence on enemas for having a bowel movement. Taking a bulk-producing agent daily Taking stool softeners daily Taking milk of magnesia every night at bedtime Self-administering a Fleet's enema every third day 65 Which laboratory finding is most indicative of a diagnosis of renal failure? Correct: A serum creatinine above 1.4 mg/dL indicates renal failure. Creatinine is the best indicator of renal failure because it is not significantly changed by other factors or conditions.Incorrect: Blood urea nitrogen can be elevated with many other factors, such as dehydration, corticosteroid therapy, and gastrointestinal bleeding, so by itself it is not a reliable indicator of renal failure.Incorrect: Serum sodium can be normal or decreased with acute renal failure, because damaged tubules cannot conserve sodium. By itself it is not diagnostic of renal failure.Incorrect: Serum potassium is elevated with acute renal failure because the kidneys' ability to excrete potassium is impaired, but by itself it is not diagnostic of renal failure. Serum creatinine 6mg/dL Blood urea nitrogen 25 mg/dL Serum sodium 137 mEq/L Serum potassium 5.2 mEq/L A client diagnosed with multiple myeloma has undergone 6 weeks of chemotherapy and now reports extreme weakness and fatigue. Which intervention is the nurse's highest care priority for this client at this time? Incorrect: Although rest is important, it is not the highest priority for this client at this time. Fatigue affects almost all clients who have cancer at some point, and nearly all clients who undergo chemotherapy experience fatigue. Resting is important, and for most clients will be self-initiated without specific interventions.Incorrect: An immunosuppressed client needs a diet high in protein and calories. Cancer can deplete the body of proteins and affect nutritional status, plus chemotherapy can cause nausea and vomiting, further compromising the client's nutritional reserves. In general, diets low in saturated fat are advised for most clients, however, immunosuppressed clients need additional calories and protein to promote healing and strength. The amount of fat included is not a priority

consideration at this time.Correct: Cancer treatments like radiation and chemotherapy affect immunity; the client becomes more susceptible to infection. To minimize the client's exposure to bacteria or viruses that can cause infections, the nurse must institute neutropenic precautions, including thorough hand hygiene for health care personnel, the client, and visitors, and instructing the client to avoid injury and to stay away from people and places that increase exposure to pathogens.Incorrect: Although regular exercise is important for the overall well-being of most clients, this is not the focus for this client at this time. The nurse should teach this client energy conservation. Rest periods between activities Reduced fat diet Neutropenic precautions Program of moderate exercise 67 A nurse is participating in a stroke screening and prevention day in her hospital. The single most important modifiable risk factor the nurse will screen participants for is: Correct: Hypertension is the single most important risk factor for stroke. Its impact can be reduced with lifestyle changes and pharmacologic therapy.Incorrect: The association between diet and stroke risk is not clear, but it is generally believed that a diet high in saturated fat and low in fruit and vegetable intake increases the risk of stroke.Incorrect: Stroke risk increases with age, doubling each decade after age 65, but this risk factor is not modifiable.Incorrect: Family and personal history of stroke increases stroke risk, but these factors cannot be modified. hypertension. a high-fat diet. advanced age. family history of stroke. 68 A 56-year-old client comes to an emergency department reporting an episode of chest pressure and indigestion that occurred 6 days ago. He tells the nurse that ever since he had that "bad upset stomach," he has felt fatigued nearly all of the time and short of breath from time to time. Which diagnostic test can help determine whether or not this client has had a myocardial infarction? Incorrect: Myoglobin is an oxygen-transporting molecule found in cardiac and skeletal muscle. Its levels increase within 1 to 3 hours of a myocardial injury and peak 12 hours after the onset of symptoms. However, they return to normal in about 12 hours, therefore myoglobin would not be useful in detecting a myocardial injury that occurred 6 days earlier.Incorrect: Total creatine kinase is actually three isoenzymes that increase in amount when cardiac cells are damaged. Levels increase within 3 to 6 hours of a myocardial injury and peak in 24 to 36 hours after the onset of symptoms. However, levels return to normal in about 3 days, therefore creatine kinase would not be useful in detecting a myocardial injury that occurred 6 days earlier.Correct: Troponin I is a protein found only in cardiac muscle, released when the muscle is injured or necrotic. Troponin levels rise within 3 to 4 hours of the acute myocardial injury, peak between 4 and 24 hours, and remain elevated for 1 to 3 weeks. Therefore, troponin I would be useful in detecting a

myocardial injury that occurred 6 days ago.Incorrect: AST is an enzyme whose levels rise when heart, liver, skeletal muscle, pancreas, kidneys, spleen, or lung tissue is injured. Levels increase within 12 hours of a myocardial injury and peak 24 hours after the onset of symptoms. However, levels return to normal in about 3 or 4 days, therefore AST would not be useful in detecting a myocardial injury that occurred 6 days earlier. Myoglobin Creatine kinase Troponin I Aspartate aminotransferase (AST) 69 A client with a compound fracture of the right tibia has had a long-leg fiberglass cast applied. To teach the client how to observe and manage his casted extremity at home, the nurse instructs him to: Correct: Increasing pain unrelieved by analgesic agents is a sign of neurovascular impairment, which could result in significant injury to the affected extremity if not relieved.Incorrect: Exposure to water should be avoided, but if a cast does get wet, the client can attempt to dry it with a hair dryer. However, he should only use the lowest possible temperature setting.Incorrect: The nurse must instruct the client never to insert any object under the cast to relieve itching. A smaller object can easily become trapped within the cast, exerting pressure on the skin and resulting in breakdown and infection. In addition, skin damage incurred by scratching could cause superficial abrasions or lacerations that could easily become infected.Incorrect: The nurse should teach the client to report any drainage or any increase in drainage to the provider immediately. Drainage could be a sign or hemorrhage, skin breakdown, or infection. report any increased pain in the casted leg that is unrelieved by the prescribed analgesia. blow-dry his cast after bathing or showering, using a moderate heat setting on the dryer. relieve any itching under the cast by using a soft device like a cotton swab padded with gauze. expect to see drainage on the cast up to about one inch (2.5 cm) in diameter. 70 A nurse is precepting a nurse recently hired to work in the postanesthesia care unit (PACU). The precepting nurse tells the orientee that the PACU nurses use the Aldrete scoring system to assess a client's readiness for transfer from the PACU. When computing an Aldrete score, the nurse assigns a score that reflects the client's: Incorrect: A nurse computing an Aldrete score assigns a value for circulation, but it is based on blood pressure, not on the quality of the client's peripheral pulses.Correct: A nurse computing an Aldrete score assigns a value for consciousness: 0 for no response, 1 for arousable on calling, and 2 for fully awake.Incorrect: A nurse computing an Aldrete score assigns a value for muscle activity, but not for peripheral sensation.Incorrect: While assessing pain level in the PACU is certainly important, pain severity is not a component of an Aldrete score. peripheral pulsations.

level of consciousness. peripheral sensation. amount of pain. A nurse is assessing a client recently admitted with dehydration due to severe and prolonged diarrhea. The client's serum potassium level is 2.1 mEq/L and the nurse initiates intravenous potassium supplementation as prescribed. Monitoring for which finding would be the nurse's highest priority for this client, as it would alert her to stop the potassium infusion and notify the provider immediately? Correct: Potassium can only be administered intravenously when the client has an adequate urine output. Less than 20 mL of urine for 2 hours in a row warrants stopping the potassium infusion. This is because potassium is primarily excreted by the kidneys. If renal function is suboptimal, the client's serum potassium level could rise to dangerous levels rapidly, increasing the risk of life-threatening dysrhythmias.Incorrect: A flattened or inverted T wave is a possible finding with hypokalemia, and an elevated U wave is classic with hypokalemia, However, once potassium supplementation is initiated, the nurse must monitor for signs of overcorrection of the imbalance. A peaked T wave, a flat P wave, and a wide QRS complex would reflect hyperkalemia.Incorrect: If the client experiences burning, the rate of infusion might have to be slowed and the site monitored closely. However, in the absence of signs of infiltration, this would not warrant discontinuing the infusion.Incorrect: This is a common finding with hypokalemia and, if this client is experiencing it, it should subside as potassium balance is restored. The nurse must monitor the client for signs of muscle weakness, which might reflect overcorrection of the potassium deficit. Urine output below 20 mL/hr for 2 consecutive hours Flattened T wave on the client's electrocardiogram Burning sensation at the site of administration Muscle cramping in the lower extremities 72 Which nursing intervention is a priority for a client with a pulmonary embolism who is receiving thrombolytic therapy via an infusion of urokinase? Incorrect: Vital signs should be assessed every 2 hours while the client is receiving thrombolytic therapy due to the increased risk of bleeding.Correct: Bed rest reduces the risk of bleeding, which is the most common adverse effect of thrombolytic agents.Incorrect: During thrombolytic therapy, procedures such as arterial punctures are kept to a minimum due to the risk of bleeding. Arterial blood gases should only be done when absolutely essential for evaluating gas exchange.Incorrect: During thrombolytic therapy, medications should be given via a noninvasive route or through existing intravenous lines. Intramuscular injections should be avoided due to the risk of bleeding. Checking vital signs every 4 hours Maintaining the client on bed rest Drawing an arterial blood gas sample every 4 hours Controlling chest pain with morphine sulfate IM every 3 to 4 hrs 73

When monitoring the results of laboratory tests of a client who has cirrhosis, the nurse expects to find a(n): Incorrect: Prothrombin time is prolonged with liver disease, and with severe liver cell damage. It will not return to normal with Vitamin K administration.Correct: Bilirubin levels reflect the ability of the liver to conjugate and excrete bilirubin, a by-product of the hemolysis of red blood cells. Bilirubin levels rise with liver disease and clinically reflect the client's degree of jaundice.Incorrect: Liver converts ammonia to urea. When this process is interrupted, as with liver disease or liver failure, serum ammonia levels rise.Incorrect: Albumin is normally formed in the liver. When liver function is impaired, as with cirrhosis, serum albumin levels drop. decreased prothrombin time. elevated serum bilirubin level. decreased serum ammonia level. elevated serum albumin level. 74 A nurse is teaching a group of clients newly diagnosed with type 1 diabetes mellitus about self-care. The nurse would include that the primary goal of insulin therapy is to help clients who have diabetes: Incorrect: This is the mechanism of action of biguanides, one type of oral hypoglycemic agent. Oral hypoglycemic agents work to improve the way the body produces and uses insulin and glucose. They are ineffective for clients who have type 1 diabetes.Correct: Insulin helps lower blood glucose by facilitating its movement into the cells. This reduces symptoms of hyperglycemia and lowers the risk of complications from sustained hyperglycemia. Maintaining blood glucose levels within an acceptable range (90 to 130 mg/dL before meals) is the primary goal of insulin therapy.Incorrect: This is the mechanism of action of sulfonylureas, one type of oral hypoglycemic agent. Clients who have type 1 diabetes no longer have functional pancreatic beta cells, therefore these drugs would be ineffective for them.Incorrect: Insulin therapy will not help clients consume a healthful, balanced diet. However, type, amount, and timing of food consumption and activity levels are factors that affect these clients' insulin needs. decrease the rate of production of insulin from the liver. keep their preprandial glucose levels below 130 mg/dL. increase the release of insulin from pancreatic beta cells. consume a well balanced and healthful diet. 75 Which statement made by a client following a bronchoscopy indicates an understanding of the discharge teaching the nurse provided? Incorrect: Coughing exercises improve lung expansion and help prevent atelectasis in clients receiving general anesthesia. But bronchoscopy is usually done with the client sedated and given local anesthesia, not with the client under general anesthesia. Also, coughing exercises are most effective when done every 1 to 2 hours.Incorrect: The client is kept NPO after the test until the gag reflex returns typically within a few hours. The client does not have to remain NPO for 8 hours after the procedure.Correct: Blood in the sputum (hemoptysis) following a bronchoscopy might indicate complications, such as perforation.

It should be reported to the provider immediately. If the client has had a tissue sample taken for biopsy during the bronchoscopy, however, the nurse should tell the client to expect that he might cough up a small amount of blood for a day or two. But any excessive amounts should be reported immediately.Incorrect: Sleeping on pillows improves breathing in clients who have orthopnea. Otherwise, there would be no indication following bronchoscopy for clients to sleep in a propped-up position. "I'll make sure I do my coughing exercises every 3 hours." "I'll wait for 8 hours after the test before I eat or drink anything." "I'll be sure to call the doctor if I cough up any blood." "I'll sleep on two pillows tonight so I can breathe easier." 76 A nurse is reviewing discharge instructions with a 72-year-old client who developed deep vein thrombosis (DVT) in the unaffected (right) leg after knee arthroplasty surgery. She is now on anticoagulant therapy and is ambulatory, but is limited in mobility due to her recent surgery. The nurse reviews techniques for eliminating modifiable risk factors for DVT, reminding the client to: Correct: One of the three major factors contributing to the development of DVT is venous stasis, the sluggish return of venous blood to the heart. Normal blood flow depends on the muscles of the extremities staying active. Flexing the knee and rotating the ankle are exercises she can easily do frequently while she is resting.Incorrect: When an extremity develops DVT, warm, moist compresses are applied to reduce discomfort. If this client, who is now ambulatory, develops increased discomfort, she must notify her provider immediately.Incorrect: The client should not wear constrictive clothing like girdles or garters, however, she must wear the compression stockings or support hose as prescribed by her provider. These snug-fitting garments help the legs muscles move blood toward the heart. This promotion of circulation helps reduce the risk of DVT recurrence.Incorrect: Although low-dose aspirin or ibuprofen (Advil) is often prescribed to prevent blot clot formation, this client is on anticoagulant therapy. The nurse must warn the client not to take any aspirin-containing or nonsteroidal anti-inflammatory drugs because they can potentiate the action of the anticoagulant and put her at increased risk for bleeding. flex her right knee frequently. apply cool compresses for any discomfort. wear loose, non-constricting stockings. take an aspirin or an NSAID tablet daily. 77 A nurse is developing a discharge teaching plan for a client newly diagnosed with ulcerative colitis. Which should be the focus of teaching the client how to help prevent exacerbations of this condition? Incorrect: Elemental formulas such as Vivonex or Ensure are absorbed in the small intestine, thus reducing bowel stimulation. This is an appropriate intervention for clients who are experiencing acute, severe symptoms of ulcerative colitis, but not on a daily basis, when the client's bowels are functioning appropriately.Incorrect: Anti-inflammatory

medications are prescribed for clients during acute exacerbations of ulcerative colitis. However, they are not helpful in maintaining remission.Incorrect: Clients who have ulcerative colitis should consume a low-residue diet that generates low fecal volume and helps reduce the likelihood of diarrhea. A high-fiber diet would irritate an inflamed bowel and increase fecal volume. Raw fruits and vegetables and whole-grain cereals and breads are examples of foods that would not be allowed on a low-residue diet.Correct: Cold foods increase intestinal motility and should be eliminated from the diet to prevent triggering a recurrence of symptoms. Limiting oral intake to elemental formulas Taking anti-inflammatory medications daily Consuming a diet high in fiber Eliminating cold foods from the diet 78 A client diagnosed with right-sided heart failure is having his central venous pressure monitored with a water manometer system. The nurse should immediately report to the client's provider a central venous pressure measurement of: Correct: This reading is outside the normal range for central venous pressure. A low reading is a sign of hypovolemia, while increasing pressures indicate worsening failure. Either should be reported immediately.Incorrect: This reading is within the normal range for central venous pressure and would not warrant immediate reporting to the provider. A low reading is a sign of hypovolemia, while increasing pressures indicate worsening failure. Either should be reported immediately.Incorrect: This reading, although at the upper limit, is still within the normal range for central venous pressure measured via pulmonary artery catheter and a pressure-monitoring system and would not warrant immediate reporting to the provider. However, when using a water-manometer system with a conventional intravenous fluid set-up, the reading would be in centimeters of water, not in millimeters of mercury.Incorrect: This reading is within the normal range for central venous pressure measured via pulmonary artery catheter and a pressure-monitoring system and would not warrant immediate reporting to the provider. However, when using a watermanometer system with a conventional intravenous fluid set-up, as is being used for this client, the reading would be in centimeters of water, not in millimeters of mercury. 16 cm H2O. 10 cm H2O. 8 mm Hg. 4 mm Hg. 79 A nurse is doing a routine clinical breast assessment on a client and notes an abnormal finding in one breast that indicates the need for this client to be referred to the provider for further evaluation to rule out or confirm breast cancer. Which sign could potentially reflect a breast malignancy? Incorrect: A common finding with breast cancer is a peau d'orange appearance, which is dimpled skin resembling an orange peel, related to interference with lymphatic drainage.Incorrect: Malignant breast lumps tend to be irregularly shaped, poorly defined, firm, hard, and embedded in the surrounding tissue.Correct: A spontaneous clear,

brownish, or blood-tinged nipple discharge can be a sign of breast cancer.Incorrect: Skin or nipple retraction is a sign of a breast malignancy. Shiny, taunt skin over the breast A round, soft, movable breast lump A brownish discharge from the nipple An excessively protruding nipple 80 When caring for a client who has been diagnosed with systemic lupus erythematosus (SLE), the nurse knows that it is a high priority to monitor weight and fluid intake and output when the client: Correct: There is a consequent fluid retention effect when corticosteroids are taken. It is important to prevent excessive fluid retention and possible renal failure and to detect signs of fluid retention as early as possible so that the appropriate interventions can be implemented.Incorrect: Although anorexia should be noted and monitored, it is not a primary clinical manifestation of SLE.Incorrect: It is important to observe for signs of gastric bleeding when NSAIDs are administered. Weight and fluid intake and output are not key components of monitoring NSAID therapy.Incorrect: The severity of SLE is evaluated by measuring the titer or strength per volume of the autoantibodies in the client's serum. The antinuclear antibodies (ANA) test result gives a rough estimate of severity of the disease. is receiving corticosteroid therapy. has a history of anorexia or other eating disorders. is receiving nonsteroidal anti-inflammatory drugs. is having the severity of the disease evaluated. 81 A nurse is assessing a client who reports flaccidity and impaired muscle function and was diagnosed with Bell's palsy. Select the area where the nurse would expect this client to have clinical manifestations of this disorder. Correct: Bell's palsy is an often temporary irritation of the motor branches of the facial nerve (cranial nerve VII) on one side of the face without any evidence of a cerebrovascular disorder. The cause is unknown, although it has been associated with herpes simplex infection. A client with Bell's palsy typically has flaccidity on the affected side of the face, with drooling and an inability to smile symmetrically or close the eyelid on the affected side.Incorrect: The biceps and other upper extremity muscles are not affected by Bell's palsy. Sudden paresis (paralysis) of an upper extremity, and often extending beyond the extremity on the affected side, is more likely related to a cerebrovascular accident.Incorrect: The muscles of the foot are not affected by Bell's palsy. Without proper therapy, clients on bedrest for extended periods of time can be left with atrophied muscles that result in lower-extremity weakness.Incorrect: The thigh muscles (quadriceps) are not affected by Bell's palsy. A more serious condition such as myasthenia gravis or muscular dystrophy causes weakness in this type of muscle group.Incorrect: The hand is not affected by Bell's palsy. A common condition that would cause muscle weakness of the hand is carpal tunnel syndrome or some other repetitive-motion disorder. 82

A nurse is caring for a client with a subarachnoid hemorrhage whose intracranial pressure (ICP) is increasing. Because a major goal for this client is optimizing cerebral tissue perfusion, the nurse: Incorrect: Coughing can raise ICP, so it must be discouraged. Instead, the nurse must carefully suction the client's secretions for no longer than 15 seconds at a time and only after hyperoxygenation.Incorrect: Isometric muscle contractions raise systemic blood pressure and therefore can raise ICP. They are contraindicated for this client.Incorrect: The client's head should be kept in a midline position, maintained with a cervical collar if necessary. This position helps promote venous drainage. Rotating or flexing the neck can compress the jugular veins and thus raise ICP. Correct: The Valsalva maneuver, often elicited by straining to have a bowel movement, elevates ICP and must be avoided in this client. Stool softeners are prescribed to help avoid this, and if the client is alert and able to eat, a high-fiber diet is indicated. encourages the client to cough to clear the airway. instructs the client to do isometric muscle contractions. turns the client's head to the opposite side every 2 hours. administers stool softeners as prescribed. 83 A nurse checks the blood glucose level of a 52-year-old client with type 1 diabetes mellitus who has been brought to an emergency department with a superficial head injury. The result is 48 mg/dL. The nurse anticipates which clinical signs and symptoms indicating this client's response to this low blood sugar level? (Check all that apply.) Correct: Abdominal cramps is an incorrect response. This is a manifestation of hyperglycemia. Cold, clammy skin is a correct response. Cool, clammy, pale skin is a classic manifestation of hypoglycemia. At the onset of the hypoglycemic episode, the parasympathetic nervous system is activated. This results in sweating and constriction of the peripheral vessels of the skin. Increased urination is an incorrect response. This is a manifestation of hyperglycemia. Tachycardia is a correct response. At the onset of the hypoglycemic episode, the parasympathetic nervous system is activated. This results in anxiety and tachycardia. Nausea/vomiting is an incorrect response. These are manifestations of hyperglycemia. Seizures is a correct response. Because the brain relies on glucose as its primary source of energy, the glucose-deprived brain produces manifestations and behavior related to altered cerebral function. Examples are disturbed behavior, seizures, and coma.Incorrect: Abdominal cramps is an incorrect response. This is a manifestation of hyperglycemia. Cold, clammy skin is a correct response. Cool, clammy, pale skin is a classic manifestation of hypoglycemia. At the onset of the hypoglycemic episode, the parasympathetic nervous system is activated. This results in sweating and constriction of the peripheral vessels of the skin. Increased urination is an incorrect response. This is a manifestation of hyperglycemia. Tachycardia is a correct response. At the onset of the hypoglycemic episode, the parasympathetic nervous system is activated. This results in anxiety and tachycardia. Nausea/vomiting is an incorrect response. These are manifestations of hyperglycemia. Seizures is a correct response. Because the brain relies on glucose as its primary source of energy, the glucose-deprived brain produces manifestations and behavior related to altered cerebral function. Examples are disturbed

behavior, seizures, and coma. Abdominal cramps Cold, clammy skin Increased urination Tachycardia Nausea/vomiting Seizures 84 A nurse suspects bowel perforation in a client receiving peritoneal dialysis when the effluent (drainage) is: Incorrect: Bloody effluent is common with the first few exchanges after a new catheter is inserted. It is also seen occasionally in young, menstruating women. Sometimes, more frequent exchanges are needed when blood is noted in the effluent, to prevent clots from obstructing the catheter.Correct: If the drainage is brown, most likely due to the presence of feces, bowel perforation is suspected.Incorrect: Effluent is normally colorless, light yellow, or straw-colored.Incorrect: Cloudy effluent is a sign of peritonitis, and antibiotic therapy is warranted. bloody. brown. yellow. cloudy. 85 Which action indicates that a client understands the most effective method for preventing the development of chronic bronchitis? Correct: Smoking is the primary cause of chronic bronchitis. Quitting smoking is, therefore, the primary method of prevention.Incorrect: Staying at an ideal weight for height, frame, and gender will help most people maintain optimal health in general and cope with many chronic illnesses. However, it is not a preventive strategy for chronic bronchitis.Incorrect: Getting an annual influenza vaccination is essential for a client who has chronic bronchitis, but it will not prevent the development of the disease.Incorrect: Exercising in cold, dry weather can trigger bronchospasm in clients who have chronic bronchitis, but this practice will not prevent the development of the disease. Smoking cessation Ideal weight maintenance Annual influenza vaccination Indoor exercise in cold weather 86 A nurse is caring for a postoperative client who has had a laryngectomy done to treat laryngeal cancer. The client is experiencing excessive drainage of mucus through the laryngectomy stoma. What is the nurse's priority intervention for helping to reduce mucus production in this client?

Incorrect: Coughing and taking deep breaths are important activities for mobilizing secretions in postoperative clients. For a client who has just had a laryngectomy, mucus is produced in excessive amounts and must be expectorated. However, coughing to remove mucus will not affect mucus production.Incorrect: Semi-Fowler's is the preferred position for a postoperative client after laryngectomy. However, this position is used to reduce edema and to decrease pressure on the suture lines. It will not help reduce the production of mucus.Incorrect: After a laryngectomy, most clients need gentle and frequent suctioning of the stoma or laryngectomy tube. However, this intervention will not keep mucus from being produced. Correct: As a result of laryngectomy, air passes directly into the trachea without being warmed and moistened by the upper respiratory mucosa. So, the tracheobronchial tree compensates by secreting excessive quantities of mucus. A major factor in reducing mucus formation and crusting around the laryngectomy stoma is increasing the humidity in the client's environment, either with a mechanical humidifier or a nebulizer. Encouraging the client to cough Positioning the client in semi-Fowler's Suctioning the client's stoma frequently Humidifying the client's environment 87 A client who has undergone a right below-the-knee amputation due to trauma has been fitted with a prosthetic limb. In reviewing prosthesis and stump care with the client, the nurse instructs him to: Incorrect: The nurse should instruct the client to discontinue use of the prosthesis if any skin irritation develops. Any skin breakdown must be checked by the provider before the client can resume using the prosthetic limb.Incorrect: The client must always wear a residual limb sock to prevent direct contact between the skin and the prosthetic socket.Correct: The prosthesis socket must be dried thoroughly with a clean cloth. Moisture between the socket and the stump can put the client at risk for fungal or bacterial infection and skin breakdown.Incorrect: The nurse must instruct the client to use no lotions, powders, or oil unless his provider specifically instructs him to do so. Instead, he should wash the residual limb each day with warm water and a bacteriostatic soap, rinse it and dry it thoroughly, and then expose it to air for at least 20 minutes. expect some skin irritation from the prosthesis as this is quite common at first. be sure the prosthesis is in direct contact with the residual limb to prevent slippage. make sure the prosthesis socket is completely dry before applying it to the limb. apply a moisturizing lotion or oil to the stump daily to keep the skin well conditioned. 88 A nurse is caring for a 55-year-old client diagnosed with a small bowel obstruction, who is undergoing gastric decompression via a nasogastric tube. The client's arterial blood gas results show a pH of 7.50, a PCO2 of 45, a PO2 of 92, an HCO3- of 31, and an O2

saturation of 98%. The nurse analyzes this result and concludes that the client has which acid-base imbalance? Incorrect: Respiratory acidosis is identified when the pH is below 7.35, the PCO2 is above 45 mm Hg, and the PO2 is below 80 mm Hg.Incorrect: Respiratory alkalosis correlates with a pH above 7.45, a bicarbonate below 21 mEq/L, and a PCO2 below 35 mm Hg.Incorrect: Metabolic acidosis is reflected in a pH is below 7.35, a bicarbonate below 21 mEq/L, and a PCO2 within normal limits (35 to 45 mm Hg).Correct: Metabolic alkalosis is indicated by an increased pH (above 7.45), an increased HCO3- (above 28 mEq/L), a rising PCO2 and a normal PO2. Serum potassium and calcium levels also drop with metabolic alkalosis. Respiratory acidosis Respiratory alkalosis Metabolic acidosis Metabolic alkalosis 89 A nurse is doing an initial assessment on a 24-year-old client who came to the emergency department reporting abdominal pain. The nurse identifies which finding as an indicator of acute appendicitis? Correct: Appendicitis typically presents with periumbilical pain that eventually shifts to the right lower quadrant.Incorrect: Fatty stools are seen with pancreatic disease, not with acute appendicitis.Incorrect: Serum amylase levels increase with pancreatitis and would rule out acute appendicitis.Incorrect: Pain associated with appendicitis is persistent and continuous. Intermittent pain is common with many other disorders, including irritable bowel syndrome. Periumbilical pain Steatorrhea Elevated amylase level Intermittent pain 90 A client with type 2 diabetes who takes glipizide (Glucotrol) in the morning calls the triage nurse at her primary care provider's office and reports feeling shaky, weak, and "nervous." The nurse tells the client to: Incorrect: It is unlikely to cause any harm for this client to drink some orange juice, unless the client is severely hyperglycemic and her symptoms have another cause, such as anxiety. This measure might or might not help resolve this client's symptoms.Incorrect: It certainly wouldn't cause this client any harm to relax, as her symptoms could indeed be a result of anxiety. However, if there is a metabolic cause for these symptoms, relaxation is unlikely to help resolve them.Incorrect: This client might or might not be taking insulin in addition to the oral hypoglycemic agent. If she is not on insulin, she would not have it available. If she is on insulin, without knowing what the client has eaten and how long ago, as well as other assessment data, it could be extremely dangerous for this client to take additional insulin.Correct: Sulfonylureas lower blood glucose levels, and these are symptoms of hypoglycemia. However, anxiety and other problems such as hyperthyroidism also have these manifestations. The first step is to determine if it is indeed hypoglycemia

that is causing this client's problem. She should check her blood glucose level immediately. drink 4 ounces of orange juice. try a relaxation exercise. take an additional dose of insulin. perform glucose self-monitoring.

Vous aimerez peut-être aussi